Sei sulla pagina 1di 103

FUNDAMENTALS OF POWER PLANT ENGINEERING

THERMODYNAMICS
1. An air bubble rises from the bottom of a well where the temperature is 25 ºC, to the surface where
the temperature is 27 ºC. Find the percent increase in the volume of the bubble if the depth of the
well is 5 m. Atmospheric pressure is 101,528 Pascals. Answer: A
A. 49.3
B. 56.7
C. 41.3
D. 38.6

2. An ideal gas is 45 psig and 80 ºF is heated in a closed container to 130 ºF. What is the final
pressure? Answer: B
A. 54 psia
B. 65 psia
C. 75 psia
D. 43 psia

3. A large mining company was provided with a 3000 cm 3 of compressed air tank. Air pressure in the
tank drops from 700 kPa to 180 kPa while the temperature remains unchanged at 28 ºC. What
percentage has the mass of air in the tank been reduced? Answer: A
A. 74
B. 72
C. 76
D. 78

4. How much work is necessary to compress air in an insulated cylinder from 0.20 m 3 to 0.01 m3. Use
T1 = 20 ºC and P2 = 100 kPa. Answer: D
A. 113.4 kJ
B. 121.4 kJ
C. 110.1 kJ
D. 115.7 kJ

5. A heat engine is operated between temperature limits of 1370 ºC and 260 ºC. Engine is supplied
with 14,142kJ/kwh. Find the Carnot cycle efficiency in percent. Answer: C
A. 70.10
B. 65.05
C. 67.56
D. 69.32

6. A closed vessel contains air at a pressure of 160 kN/m 2 gauge and temperature of 30 ºC. The air is
heated at constant volume to 60 ºC with the atmospheric pressure of 759 mm Hg. What is the final
gauge pressure? Answer: B
A. 174
B. 186
C. 167
D. 172
7. What is the temperature in ºC of 2 liters of water at 30 ºC after 500 Calories of heat have been
added? Answer: B
A. 35.70
B. 30.2
C. 38
D. 39.75

8. A volume of 450 cm3 of air is measured at a pressure of 740 mm Hg absolute and a temperature of
20 ºC. What is the volume in cm3 at 760 mm Hg absolute and 0 ºC? Answer: B
A. 516.12
B. 408.25
C. 620.76
D. 375.85

9. Assuming compression is according to the law PV=constant. Calculate the initial volume of gas at
a pressure of 2 bar which will occupy a volume of 6 m 3 when it is compressed to a pressure of 42
bar. Answer: A
A. 1263
B. 1203
C. 1303
D. 1363

10. A steam condenser receives 10 kg per second of steam with an enthalpy of 2,570kJ/kg. Steam
condenses into liquid and leaves with an enthalpy of 160kJ/kg. Cooling water passes through the
condenser with temperature increases from 13 ºC to 24 ºC. Calculate the cooling water flowrate in
kg/s.
A. 533
B. 518
C. 523
D. 528

11. A steam expands adiabatically in a turbine from 2,000 kPa, 400 ºC to 400 kPa, 250 ºC. What is the
effectiveness og the process in percent assuming an atmospheric temperature of 15 ºC. Neglect
changes in kinetic and potential energy. Answer: C
A. 82
B. 84
C. 80
D. 86

12. Steam enters the superheaters of a boiler at a pressure of 25 bar and dryness of 0.98 and leaves at
the same pressure at a temperature of 370 ºC. Calculate the heat energy supplied in the suerheaters.
Properties of steam:
At 25 bar and 370 ºC (h= 3171.8kJ/kg) and at 25 bar and 0.98 dryness (h f =962.11kJ/kg, hfg
=1841.01kJ/kg).Answer: C
A. 407.46
B. 408.57
C. 405.51
D. 406.54
13. The thermal efficiency of a particular engine operating on an ideal cycle is 35%. Calculate the heat
supplied per 1200 watt-hr of work developed in kJ. Answer: A
A. 12,343
B. 10,216
C. 14,218
D. 11,108

14. Determine the average Cp value in kJ/kg-K of a gas if 522 kJ of heat is necessary to raise the
temperature from 300 K to 800 K making the pressure constant. Answer: B
A. 1.440
B. 1.044
C. 1.038
D. 1.026

15. Determine the force in Newtons required to produce an acceleration of 0.30 m/s 2 on a 0.05 kg
mass. Answer: C
A. 0.018 N
B. 0.025 N
C. 0.015 N
D. 0.200 N

16. Ammonia weighing 22 kg is confined inside a cylinder equipped with a piston has an initial
pressure of 413 kPa at 38 ºC. If 2900 kJ of heat is added to the ammonia until its pressure and
temperature are 413 kPa and 100 ºC, respectively. What is the amount of work done by the fluid in
kJ? Answer: A
A. 667
B. 304
C. 420
D. 502

17. What is the total required heating energy in raising the temperature of a given amount of water
when the energy supplied is 1000 kwh with heat losses of 25%? Answer: C
A. 1000
B. 1500
C. 1333
D. 1250

18. A vessel of 0.058 m3 capacity is well insulated and is divided equally by rigid conducting
diaphragm. Initially both halves contain air at pressure of 137.8 kPa and 413.4 kPa and a
temperature of 27 ºC and 177 ºC respectively. What is the increase of entropy of the system in kJ/
ºC. Answer: C
A. 1.002
B. 0.5080
C. 0.00173
D. 0.1080
19. A certain gas at 101.325 kPa and 16 ºC whose volume is 2.83 m 3 are compressed into a storage
vessel of 0.31 m3 capacity. Before admission, the storage vessel contained the gas at a pressure and
temperature of 137.8 kPa and 24 ºC. after admission, the pressure has increased to 1171.8 kPa.
What should be final temperature of the gas in the vessel in Kelvin? Answer: B
A. 298.0
B. 319.0
C. 180
D. 420

20. Steam enters a throttling calorimeter at a pressure of 1.03 Mpa. The calorimeter downstream
pressure and temperature are respectively 0.100 Mpa and 125 ºC. what is the percentage moisture
of the supply steam?
Properties of steam:
P= 1.03 Mpa hg = 2279.25kJ/kg hfg= 2010.7 kJ/kg
At 0.100 Mpa and 125 ºC: h= 2726.6kJ/kg. Answer: A
A. 2.26
B. 5.21
C. 3.15
D. 1.98

21. What pressure is column of water 100 cm high equivalent to? Answer: B
A. 9807 dynes/cm2
B. 9807 N/m2
C. 0.10 bar
D. 98100 N/m2

22. A water temperature rise of 18 ºF in the water cooled condenser is equivalent in ºC to: Answer: C
A. -9.44 ºC
B. 263.56 ºC
C. 10 ºC
D. 7.78 ºC

23. Steam flows into a turbine at the rate of 10 kg/s and 10 kW of heat are lost from the turbine.
Ignoring elevation and kinetic energy effects, calculate the power output from the turbine.
Note: h1= 2739.0 kJ/kg and h2= 2300.5 kJ/kg. Answer: C
A. 4605 kW
B. 4973 kW
C. 4375 kW
D. 4000 kW

24. The enthalpy of air is increased by 139.586 kJ/kg in a compressor. The rate of air flow is 16.42
kg/min. the power input is 48.2 kW. Which of the following values most nearly equals the heat loss
from the compressor in kW? Answer: A
A. -10
B. 10.2
C. -999.95
D. 9.95
25. A Carnot engine requires 35 kJ/s from the hot source. The engine produces 15 kW of power and
temperature of the sink is 26ºC.What is the temperature of the hot source in ºC? Answer: C
A. 245.57
B. 210.10
C. 250.18
D. 260.68

26. The maximum thermal efficiency possible for a power cycle operating between 1200 ºF and 225 ºF
is: Answer: B
A. 58%
B. 8.73%
C. 57.54%
D. 57.40%

27. A Carnot engine receives 130 Btu of heat from a hot reservoir at 700 ºF and rejects 49 Btu of heat.
Calculate the temperature of the cold reservoir. Answer: D
A. -21.9 ºF
B. -24.2 ºF
C. -20.8 ºF
D. -22.7 ºF

28. A tank contains 80 ft3 of air at a pressure of 350 psi. if the air is cooled until its pressure and
temperature decrease to 200 psi and 70 ºF respectively, what is the decrease in internal energy?
Answer: B
A. 4575
B. -5507
C. 5552
D. 0

29. If 10 lbs of water evaporated at atmospheric pressure until a volume of 288.5 ft 3 is occupied, how
much work is done? Answer: A
A. 610,358 ft-lb
B. 0
C. -610,538 ft-lb
D. 550,000 ft-lb

30. A reverse Carnot cycle requires 3 Hp and extracts energy from a lake to heat a house. If the house
is kept at 70 ºF and requires 2000 Btu per minute, what is the temperature of the lake? Answer: B
A. 35 ºF
B. 36 ºF
C. 39 ºF
D. 40 ºF

31. A pressure gage registers 50 psig in a region where the barometer is 14.25 psia. Find the absolute
pressure in psia. Answer: A
A. 64.25
B. 443
C. 151.325
D. 35.75
32. A pump discharges 50 tons of water per hour to a height of 8 m, the overall efficiency of the
pumping system being 69%. Calculate the output and input power. Answer: A
A. 1.09 kW & 1.58 kW
B. 2.09 kW & 2.58 kW
C. 1.09 kW & 2.58 kW
D. 2.09 kW & 1.58 kW

33. A condenser vacuum gauge reads 715 mm Hg when the barometer stands at 757 mm Hg. What is
the absolute pressure in the condenser in kPa. Answer: C
A. 196.25 kPa
B. 100.92 kPa
C. 5.60 kPa
D. 95.33 kPa

34. Oil flows full bore at a velocity of 2 m/s through a nest of 16 tubes in a single pass cooler. The
internal diameter of the tubes is 30 mm and the density of the oil is 0.85 g/li. Find the volume flow
in liters per second. Answer: D
A. 20
B. 21
C. 22
D. 23

35. Calculate the quantity of heat to be transferred to 2.25 kg of brass to raise its temperature from 20
ºC to 240 ºC taking the specific heat of the brass as 0.394 kJ/kg-K. Answer: C
A. 519.03 kJ
B. 159.03 kJ
C. 195.03 kJ
D. 591.03 kJ

36. Three cubic meter of air at a pressure of 20 kPag is compressed at constant temperature to a
pressure as 425 kPag. Calculate the final volume of the air.
A. 0.80 m3
B. 0.69 m3
C. 0.92 m-3
D. 0.96 m3

37. The pressure of the air in a starting air vessel is 40 bar and the temperature is 24 ºC. If a fire in the
vicinity causes the temperature to rise to 65 ºC, find the pressure of the air. Neglect any increase in
volume of the vessel. Answer: A
A. 45.52 bar
B. 54.52 bar
C. 42.55 bar
D. 44.55 bar

38. One cubic meter of a perfect gas at a constant pressure of 1 bar and temperature of 17 ºC are
compressed to a volume of 0.50 m 3 and the final pressure is 6 bar. Calculate the final temperature.
Answer: C
A. 870 ºC
B. 560 ºC
C. 597 ºC
D. 860 ºC
39. An air compressor delivers 0.20 m3 of air at a pressure of 850 kPa and 31 ºC into an air reservoir.
Taking the gas constant of air as 0.287 kJ/kg-K, calculate the mass of air delivered. Answer: B
A. 1.59 kg
B. 1.95 kg
C. 5.91 kg
D. 5.19 kg

40. An oxygen cylinder of volume 2.3 ft 3 has a pressure of 2200 psig and is at 70 ºF. determine the
mass of oxygen in the cylinder. Answer: C
A. 26.66 lbs
B. 26.86 lbs
C. 28.66 lbs
D. 28.88 lbs

41. What is the power of a steam jet 15 mm in diameter moving at 750 m/s? Steam condition, 1.4
kg/cm2, dry and saturated (v= 1.256 m3/kg). Answer: A
A. 29.67 kW
B. 39.67 kW
C. 19.67 kW
D. 49.67 kW

42. How much power is there in the kinetic energy of the atmosphere (density = 1.217 kg/m 3) at 56 kph
wind velocity? Consider the section that passes through a 3 m diameter circular area normal to the
velocity. Answer: C
A. 10.21 kW
B. 15.21 kW
C. 16.21 kW
D. 17.21 kW

43. An electric heater is to heat 11 kg of oil per min. from 4.5 ºC to 65.5 ºC. Specific heat of the oil is
2.1 J/gm- ºC. How many watts should this heater consume? Answer: D
A. 20,448 W
B. 33,448 W
C. 20,400 W
D. 23,442 W

44. Find the enthalpy of 1 kg of wet saturated steam at a pressure of 0.20 bar and dryness fraction of
0.85. At 0.20 bar, hf= 251 kJ/kg & hfg= 2358 kJ/kg. Answer: C
A. 2200 kJ/kg
B. 2525.30 kJ/kg
C. 2255.30 kJ/kg
D. 2000 kJ/kg

45. If the specific enthalpy of wet saturated steam at a pressure of 11 bar is 2681 kJ/kg, find its dryness
fraction. At 11 bar, hf= 781 kJ/kg & hfg= 2000 kj/kg. Answer:C
A. 0.75
B. 0.85
C. 0.95
D. 0.65
46. Find the heat transfer required to convert 5 kg of water at a pressure of 20 bar and temperature of
21 ºC into steam of dryness fraction 0.90 at the same pressure.
Properties of steam: P= 20 bar; hf = 909 kJ/kg; hfg= 1890 kJ/kg
Note: for water at 21 ºC, h= 88 kJ/kg. Answer: C
A. 10,610 kJ
B. 11,610 kJ
C. 12,610 kJ
D. 15,610 kJ

47. A closed vessel contain air at a pressure of 140 Kpag and temperature of 20 ºC. Find the final
gauge pressure if the air is heated at constant volume to 40 ºC. Take the atmospheric pressure as
759 mm Hg.
A. 156.46
B. 146.46
C. 136.46
D. 126.46

48. Water substance at 70 bar and 65 ºC enters a boiler tube of constant inside diameter of 25 mm. The
water leaves the boiler tube at 50 bar and 700 K at velocity of 100 m/s. Calculate the inlet volume
flow in li./sec. At 70 bar & 65 ºC, v1 = 0.001017 m3/kg , at 50 bar & 700 K, v2 = 0.06081 m3/kg.
Answer: C
A. 0.75
B. 0.64
C. 0.82
D. 0.96

49. A group of 50 persons attend a secret meeting in room which is 12 m wide by 10 m long, and a
ceiling height of 3 m. The room is completely sealed off and insulated. Each person gives off 150
kCal per hour of heat and occupies a volume of 0.20 m 3. The room has an initial pressure of 101.3
kPa and temperature of 16 ºC. Calculate the room temperature after 10 minutes. Use R = 0.287
kJ/kg-K and Cv = 0.171 kCal/kg-K. Answer:A
A. 33.1 ºC
B. 37.7 ºC
C. 38.7 ºC
D. 31.7 ºC

50. Steam at 1000 lbf/ ft2 pressure and 300 ºR has specific volume of 6.5 ft 3/lbm and a specific enthalpy
of 9800 ft-lbf/lbm. . Find the internal energy per pound mass of steam. Answer: B
A. 2500 ft-lbf/lbm
B. 3300 ft-lbf/lbm
C. 5400 ft-lbf/lbm
D. 6900 ft-lbf/lbm

51. Determine the average constant pressure specific heat of steam at 10 kPa and 45.8 ºC. Note: From
steam table, at 47..7 ºC, h = 2581.1 kJ/kg. Answer: A
A. 1.79 kJ/kgºC
B. 10.28 kJ/kgºC
C. 30.57 kJ/kgºC
D. 100.1 kJ/kgºC
52. A 10 m3 vessel initially contains 5 m 3 of saturated vapor at 100 kPa. Calculate the internal energy
of the system.
Properties of liquid water and saturated vapor: At 100 kPa, v f = 0.001043 m3/kg; vg = 1.6940
m3/kg; uf = 417.3 kJ/kg; ug = 2506.1 kJ/kg. Answer: A
A. 2.0 x 106 kJ
B. 1.0 x106 kJ
C. 3.0 x 106 kJ
D. 5.0 x 106 kJ

53. A vessel with a volume of 1 m 3 contains liquid water and water vapor in equilibrium at 600 kPa.
The liquid water has a mass of 1 kg. Calculate the mass of the water vapor.
Properties of liquid water and water vapor at 600 kPa: v f = 0.001101 m3/kg vg = 0.3157 m3/kg.
Answer: D
A. 1.57 kg
B. 1.89 kg
C. 2.54 kg
D. 3.16 kg

54. If 6 liters of a gas at a pressure of 100 kPaa are compressed reversibly according to PV 2 = C, until
the volume becomes 2 liters. Find the final pressure. Answer: C
A. 600 kPaa
B. 800 kPaa
C. 900 kPaa
D. 1000 kPaa

55. The flow energy of 124 liters per minute of a fluid passing a boundary to system is 108.5 kJ/min.
Determine the pressure at this point. Answer: A
A. 875 kPa
B. 675 kPa
C. 975 kPa
D. 575 kPa

56. Work done by a substance in reversible non-flow manner in accordance with V = 100/p ft 3, where P
is in psia. Evaluate the work done on or by the substance as the pressure increases from 10 psia to
100 psia. Answer: B
A. 33,157.22 ft-lb
B. -33,157.22 ft-lb
C. 43,157.22 ft-lb
D. -43,157.22 ft-lb

57. A closed gaseous system undergoes a reversible process during which 25 Btu are rejected, the
volume changing from 5ft3 to 2 ft3, and the pressure remains constant at 50 psia. Find the change of
internal energy. Answer: C
A. -52.76 Btu
B. -2.76 Btu
C. 2.76 Btu
D. 52.76 Btu
58. Assume 8 lb of a substance receive 240 Btu of heat at constant volume and undergo a temperature
change of 150 ºF. Determine the average specific heat of the substance during the process. Answer:
D
A. 0.5 Btu/lb ºF
B. 0.3 Btu/lb ºF
C. 0.40 Btu/lb ºF
D. 0.20 Btu/lb ºF

59. The following expressions relate to a particular gaseous mass: PV= 95T, h= 120 + 0.60T where
these units obtain in psf, V in ft3/lb, T in ºR and h in Btu/lb. If the specific heats are temperature
dependent only, find Cp and Cv. Answer: A
A. 0.6 Btu/lb ºR, 0.48 Btu/lb ºR
B. 0.5 Btu/lb ºR, 0.50 Btu/lb ºR
C. 0.60 Btu/lb ºR, 0.7 Btu/lb ºR
D. 0.50 Btu/lb ºR, 0.48 Btu/lb ºR

60. Calculate the entropy of steam at 60 psia with a quality of 0.60. properties of steam at 60 psia: S f =
0.4274 Btu/lb ºR. Answer: B
A. 0.4247 Btu/lb ºR
B. 1.1577 Btu/lb ºR
C. 0.7303 Btu/lb ºR
D. 0.896 Btu/lb ºR

61. Find the change in internal energy of 5 lbm of oxygen when the temperature changes from 100 ºC to
120 ºC, Cv = 0.157 Btu/lbm- ºR. Answer: B
A. 14.70 Btu
B. 15.70 Btu
C. 16.80 Btu
D. 147 Btu

62. Water (Specific heat, Cv =4.2kJ/kg-K) is being heated by 1500- W heater. What is the rate of
change in temperature of 1 kg of water? Answer: C
A. 0.043 Kelvin/s
B. 0.179 Kelvin/s
C. 0.357 Kelvin/s
D. 1.50 Kelvin/s

63. One kilogram of water (Cv =4.2 kJ/kg-K) is heated by 300 Btu of energy. What is the change in
temperature in K? Answer: A
A. 75.36 K
B. 125.20 K
C. 73.80 K
D. 17.96 K

64. Determine the change in enthalpy per lb mass of nitrogen gas as its temperature changes from 500
ºF to 200 ºF. (Cp= 0.2483 BTU/lbm ºR). Answer: A
A. -74.49 Btu/lbm
B. -72.68 Btu/lbm
C. -68.47 Btu/lbm
D. -63.78 Btu/lbm
65. Calculate the change in enthalpy as 1 kg of nitrogen is heated from 1000 K to 1500 K, assuming
the nitrogen is an ideal gas at a constant pressure. The temperature dependent specific heat of
nitrogen is Cp = 39.06 – 512.79 T-1.5 + 1072.7 T-2 – 820.4 T-3 where Cp is in kJ/kg-mol, and T is in
K. Answer: B
A. 600 kJ
B. 697.27kJ
C. 800 kJ
D. 897.27 kJ

66. What is the resulting pressure when one pound of air at 15 psia and 200 ºF is heated at constant
volume to 800 ºF?
A. 15 psia
B. 28.6 psia
C. 36.4 psia
D. 52.1 psia

67. The temperature of an ideal gas remains constant while the absolute pressure changes from 103.4
kPaa to 827.2 kPaa. If the initial volume is 80 liters, what is the final volume? Answer: B
A. 5 li
B. 10 li
C. 15 li
D. 20 li

68. For a certain ideal gas, R= 0.277 kJ/kg-K and k= 1.384. What are the values of C p and Cv? Answer:
A
A. 0.9984 kJ/kg-K, 0.7213 kJ/kg-K
B. 1 kJ/kg-K, 0.8124 kJ/kg-K
C. 0.7124 kJ/kg-K, 0.8124 kJ/kg-K
D. 0.9984 kJ/kg-K, 0.6124 kJ/kg-K

69. A mixture is formed at 689.48 kPaa, 37.8 ºC by bringing together these gases each volume before
mixing measured at 689.48 kPaa, 37.8 ºC; 3 mol CO 2, 2 mol N2, 4.5 mol O2. Find the partial
pressure of CO2 after mixing. Answer: A
A. 217.73 kPaa
B. 145.15 kPaa
C. 326.60 kPaa
D. 445.15 kPaa

70. An air with mass of 0.454 kg and an unknown mass of CO 2 occupy an 85 liters tank at 2068 Kpaa.
If the partial pressure of the CO2 is 344.74 kPaa, determine its mass. Answer: A
A. 0.138 kg
B. 0.238 kg
C. 0.183 kg
D. 0.283 kg

71. After series of state changes, the pressure and volume of 2.286 kg of Nitrogen are each doubled.
What is ΔS? Answer: A
A. 2.807 kJ/kg-K
B. 2.268 kJ/kg-K
C. 2.987 kJ/kg-K
D. 3.407 kJ/kg-k
72. The temperature of 4.82 lb of Oxygen occupying 8 ft 3 is changed from 110 ºF to 200 ºF while
pressure remains constant at 115 psia. Determine the final volume. Answer: C
A. 7.26 ft3
B. 8.26 ft3
C. 9.26 ft3
D. 10.26 ft3

73. Twenty grams of oxygen gas (O 2) are compressed at a constant temperature of 30 ºC to 5% of their
original volume. What work is done on the system? Use R of air, 0.0619 Cal/gm-K. Answer: D
A. 824 Cal
B. 924 Cal
C. 944 Cal
D. 1124 Cal

74. Helium (R = 0.4968 Btu/lbm ºR) is compressed isothermally from 14.7 psia and 68 ºF. The
compression ratio is 4. Calculate the work done by the gas. Answer: A
A. -364 Btu/lbm
B. -145 Btu/lbm
C. -187 Btu/lbm
D. -46.7 Btu/lbm

75. Gas is enclosed in a cylinder with a weighed piston as the top boundary. The gas is heated and
expands from a volume of 0.04 m3 at a constant pressure of 200 kpa. Calculate the work done by
the system. Answer: C
A. 8 kJ
B. 10 kJ
C. 12 kJ
D. 14 kJ

76. A piston- cylinder system contains a gas which expands under a constant pressure of 1200 lb/ft 2. If
the piston is displaced 1 ft during the process, and the piston diameter is 2 ft. what is the work done
by the gas on the piston?
A. 1768 ft-lb
B. 2387 ft-lb
C. 3768 ft-lb
D. 4000 ft-lb

77. Gas enclosed in a cylinder with a weighted piston as the top boundary. The gas is heated and
expands from a volume of 0.04 m3 to 0.10 m3. The pressure varies such that PV = constant and the
initial pressure is 200 kPa. Calculate the work done by the system. Answer: B
A. 6.80 kJ
B. 7.33 kJ
C. 9.59 kJ
D. 12 kJ
78. In an isentropic process, P1 = 200 psi, P2 = 300 psi and T1 = 700 ºR. Find T2 using k= 1.4. Answer:
C
A. 576 ºR
B. 680 ºR
C. 786 ºR
D. 590 ºR

79. Nitrogen (k= 1.4) is expanded isentropically. Its temperature changes from 620 ºF to 60 ºF. Find the
pressure ratio (P1/P2). Answer: B
A. 0.08
B. 12.91
C. 26.2
D. 35.47

80. Nitrogen is expanded isentropically. Its temperature changes from 620 ºF to 60 ºF. The volumetric
ratio is (V1/V2) = 6.22 and the value of R for nitrogen is 0.0787 Btu/lb mºR. What is the work done
by the gas? Answer: A
A. -110.18 Btu/lbm
B. 120.27 Btu/lbm
C. 110.18 Btu/lbm
D. -120.27 Btu/lbm

81. If the ºF scale is twice the ºC scale, what is the reading in the Fahrenheit scale? Answer: B
A. 160 ºC
B. 320 ºC
C. 140 ºC
D. 280 ºC

82. Water enters the condenser at 30 ºC and leaves at 60 ºC.What is the temperature difference in ºF?
Answer: C
A. 16.67
B. 48.67
C. 54
D. 22

83. A cylinder and piston arrangement contains saturated water vapor at 110 ºC. The vapor is
compressed in a reversible adiabatic process until the pressure is 1.6 Mpa. Determine the work
done by the system per kg of water. At 110 ºC, S 1 = 7.2387 kJ/kg-K, U1 = 2518.1 kJ/kg and at 1.6
Mpa, S2 = 7.2374 kJ/kg-K, U2 = 2950.1 kJ/kg-K, T2 = 400 ºC. Answer: B
A. -500 kJ/kg
B. -432 kJ/kg
C. -632 kJ/kg
D. -700 kJ/kg

84. Helium is compressed isothermally from 14.7 psia and 68 ºF. The compression ratio is 4. Calculate
the change in entropy of the gas given that RHelium = 0.4961 Btu/lbm ºR. Answer: A
A. -0.688 Btu/lbm ºR
B. -2.76 Btu/lbm ºR
C. 0.688 Btu/lbm ºR
D. 2.76 Btu/lbm ºR
85. Steam at a pressure of 9 bar (h f = 2031 kJ/kg) is generated in an exhaust gas boiler from feedwater
at 80 ºC (h = 334.9 kJ/kg). if the dryness fraction of the steam is 0.96, Determine the heat transfer
per kilogram of steam. Answer: A
A. 2357.86
B. 3357.86
C. 1357.86
D. 5357.86

86. If wet saturated steam at 8 bar (h fg = 2048 kJ/kg) requires 82 kJ of heat per kg of steam to
completely dry it, what is the dryness fraction of the steam? Answer: C
A. 0.76
B. 0.86
C. 0.96
D. 0.66

87. Wet saturated steam at 17 bar (h f = 872 kJ/kg, hfg = 1293 kJ/kg ) dryness 0.97 is produced from
feedwater at 85 ºC (h = 335.9 kJ/kg). Find the heat energy supplied per kg. Answer: C
A. 4381.41 kJ/kg
B. 1381.41 kJ/kg
C. 2381.41 kj/kg
D. 3381.41 kJ/kg

88. A turbo-generator is supplied with superheated steam at a pressure of 30 bar and temperature 350
ºC (h = 3117 kJ/kg). The pressure of the exhaust steam from the turbine is 0.06 bar (h f = 15.2 kJ/kg,
hfg = 2415 kJ/kg) with a dryness fraction of 0.88. If the turbine uses 0.25 kg per second, calculate
the power equivalent of the total enthalpy drop. Answer: B
A. 109.95 kW
B. 209.95 kW
C. 309.95 kW
D. 409.95 kW

89. Steam enters the superheaters of a boiler at a pressure of 20 bar (h f = 909 kJ/kg, hfg = 1890kJ/kg, v
= 0.09957 m3/kg) and dryness 0.98 and leaves at the same pressure at a temperature of 350 ºC (h =
3138 kJ/kg, v = 0.01386 m3/kg). Find the percentage increase in volume due to drying and
superheating. Answer: D
A. 12.04
B. 22.04
C. 32.04
D. 42.04

90. Steam at the rate of 500 kg/hr is produced by a steady flow system boiler from feedwater entering
at 40 ºC. Find the rate at which heat is transformed in kCal/kg and of steam 50 kCal/kg. Answer: A
A. 275,000 kCal/hr
B. 175,000 kCal/hr
C. 375,000 kCal/hr
D. 475,000 kCal/hr
91. Stean leaves an industrial boiler at 827.4 kPa and 171.6 ºC (h f = 727.25 kJ/kg, hfg = 2043.2 kJ/kg).
A portion of the steam is passed through a throttling calorimeter and is exhausted to the atmosphere
when the calorimetr pressure is 101.4 kpa and a temperature of 115.6 ºC (h = 2707.6 kJ/kg). How
much moisture does the steam leaving the boiler contain? Answer: B
A. 2.08%
B. 3.08%
C. 4.08%
D. 5.08%

92. During the polytropic process of an ideal gas, the state changes from 138 kpa and 5 ºC to 827 kPa
and 171 ºC. Find the value of n. Answer: A
A. 1.354
B. 1.253
C. 1.345
D. 1.234

93. For an ideal gas, what is the specific molar entropy change during an isothermal process in which
the pressure changes from 200 kPa to 150 kPa? Answer: A
A. 2.39 J/mol-K
B. 1.39 J/kg-K
C. 3.39 J/kg-K
D. 4.39 J/kg- K

94. Water enters the heater at 25 ºC and leaves at 80 ºC. What is the temperature change in ºF?
Answer: B
A. 55
B. 99
C. 11
D. 65

95. The suction pressure of a pump reads 600 mm hg vacuum. What is the absolute pressure in kPa?
Answer: C
A. 11.33
B. 31.33
C. 21.33
D. 41.33

96. One kilogram of wet steam at a pressure of 8 bar (vg = 0.2404 m3/kg, vf = 0.0011148 m3/kg) and
dryness 0.94 is expanded until the pressure is 4 bar (vg = 0.4625 m3/kg, vf = 0.0010836 m3/kg). If
expansion follows the law PVn = C, where n= 1.12, find the dryness fraction of the steam at the
lower pressure. Answer: A
A. 0.9072
B. 0.4197
C. 0.2260
D. 0.2404
97. 2.5 liters oof superheated steam at 25 bar and 400 ºC (v = 0.1252m3/kg) is expanded in an engine to
a pressure of 0.1 bar (vg = 14.674 m3/kg, vf = 0.0010102 m3/kg) when its dryness fraction is 0.9.
Find the final volume of the steam. Answer: A
A. 163.74 liters
B. 263.74 liters
C. 363.74 liters
D. 463.74 liters

98. A 1.5 kg of wet steam at a pressure of 5 bar (h f = 640 kJ/kg, hfg = 2109 kJ/kg) dryness 0.95 is blown
into 70 liters of water 12 ºC (h = 50.4 kJ/kg). Find the final enthalpy of the mixture. Answer: D
A. 74.80 kj/kg
B. 84.80 kJ/kg
C. 94.80 kJ/kg
D. 104 kJ/kg

99. Wet saturated steam at 16 bar (h f = 859 kJ/kg, hfg = 1935 kj/kg) reducing valve and is throttled to a
pressure of 8 bar (hf = 721 kJ/kg, hfg = 2084 kJ/kg). Find the dryness fraction of the reduced
pressure steam. Answer: C
A. 0.8833
B. 0.7733
C. 0.9933
D. 0.6633

100. A vessel of volume8.7 m3 contains air and dry saturated at a total pressure of 0.06 bar and
temperature 29 ºC (Psat = 0.04 bar, v = 34.80 m3/kg). Taking R for air as 287 J/kg-K, calculate the
mass of steam and the mass of air in the vessel. Answer: A
A. 0.25 kg, 0.204 kg
B. 0.35 kg, 0.204 kg
C. 0.25 kg, 350 kg
D. 0.35 kg, 0.45 kg

PROBLEMS IN FLUID MECHANICS

1. A fluid that has a velocity of 18m/s have an equivalent head of: Answer: A
A. 16.51 m
B. 13.56 m
C. 18.34 m
D. 11.41 m

2. The length of pipe is 168 meters. If the pressure drop is 50 kPa for every 30 m, what is the total
pressure drop? Answer: C
A. 240 kPa
B. 260 kPa
C. 280 kPa
D. 220 kPa
3. Find the volume flow rate of water in L/s if the power developed under a head of 320 m is 10,500
kW and the hydraulic efficiency is 85%. Answer: A
A. 3,935
B. 3,293
C. 3,271
D. 3,721

4. Water flows in a pipe with a velocity of 10 m/s. determine the velocity head of the water . Answer:
A
A. 5.1 m
B. 6.8 m
C. 1.5 m
D. 8.6 m

5. A cylindrical pipe with water flowing downward at 0.02 m 3/s having top diameter of 0.08, bottom
diameter of 0.04 m and height of 1.5 m. find the pressure between the pipe. Answer: B
A. 94.05 kPa
B. 104.06 kPa
C. 124.07 kPa
D. 134.10 kPa

6. In a hydraulic press the large piston has a cross- sectional area A 1 = 200 cm2 and the small piston
has a cross- sectional area A2 = 5 cm2. If the force applied is 250 N to the small piston. Compute
the force acting on the large piston. Answer: A
A. 10 kN
B. 12 kN
C. 21 kN
D. 20 kN

7. A solid material in a cube shape floats in oil of density 800 kg/m 3 with one- third of the block out
of the oil. What is the buoyant force on the cube whose edge is 0.75 cm. Answer: A
A. 2.207 x 10-3 N
B. 2.581 x 10-3 N
C. 1.255 x 10-3 N
D. 3.012 x 10-3 N

8. A balloon having a mass of 400 kg remains suspended motionless in the air. If the air density is
1.29 kg/m3, what is the volume of the balloon in cubic meters? Answer: A
A. 310 m3
B. 340 m3
C. 280 m3
D. 270 m3

9. A block of wood that weighs 71.2 N and of specific gravity 0.72 is tied by a string to the bottom of
a tank of water in order to have the block totally immersed. Determine the tension in the string.
Answer: A
A. 23.73 N
B. 47.72 N
C. 94.93 N
D. 50.25 N
10. A cylindrical wooden buoy of height 3 m and mass 80 kg, floats vertically in water. If its specific
gravity is 0.8, how much will it be depressed when a body of mass 10 kg is placed on its upper
surface? Answer: A
A. 0.30 m
B. 0.40 m
C. 0.35 m
D. 0.45 m

11. The weight of the balloon and the gas it contains is 11.12 kN. If the balloon displaced 1132 m 3 of
air and weight of 12.3 N, what is the acceleration with which the balloon begins to rise? Answer: A
A. 2.47 m/s2
B. 3.76 m/s2
C. 2.0 m/s2
D. 3.24 m/s2

12. During a heavy rain, a 3 m x 4.6 m room is flooded to a depth of 15 cm. To remove the water (µ =
1.0 x 10-3 Pa.s), a pump is used that does the job in two hours. The water flows through a horizontal
pipe of radius 0.64 cm and length of 6.7 m. Compute the gage pressure does the pump produce?
Answer: A
A. 2.923 kPa
B. 2.785 kPa
C. 1.176 kPa
D. 1.254 kPa

13. If the velocity of the flow in a 75- mm diameter fire hose is 25 cm/s, what is the power available in
a 30-mm diameter jet issuing from a nozzle attached at the end of the pipe. Answer: A
A. 1.34 W
B. 3.24 W
C. 1.25 W
D. 2.34 W

14. A mercury barometer stands 762 mm. A gas bubble, whose volume is 33 cm 3 when it is at the
bottom of the lake 45.7 m deep rises to the surface. Compute the volume at the surface of the lake.
Answer: A
A. 178 cm3
B. 258 cm3
C. 520 cm3
D. 320 cm3

15. A 4.9 ft3 of water are compressed to 5000 psig. What is the volume decrease if the temperature is
60 ºF? Assume the average bulk modulus of elasticity of 311000. Answer: A
A. 0.089 ft3
B. 0.099 ft3
C. 0.079 ft3
D. 0.069 ft3
16. A cylindrical tank having a diameter 1.5 m and a height 4 m is open at one end and closed at the
other end. It is placed below the water surface with its depth of the water inside the tank is 1.8 m?
Answer: C
A. 5.90cm
B. 7.0 cm
C. 6.25 cm
D. 6.0 cm

17. A 70.874 L/s (150 ft3/min.) of water is flowing in a round pipe such that the flow remains laminar.
Considering that the critical Reynold’s number to be 2000, what is the diameter of the pipe
necessary to ensure laminar flow. (Note: kinematic viscosity of water is 4.75 x 10 -5 ft2/s). Answer:
A
A. 33.50 ft
B. 41.7 ft
C. 12.5 ft
D. 37.5 ft

18. Water is flowing in a pipe of varying cross- sectional area, and at all points the water completely
fills the pipe. The cross- sectional area @ point 1 is 0.080 m 2, and the velocity is 3.5 m/s. Compute
the fluid’s velocity at point 2 where the cross- sectional area is 0.060 m 2. Answer: A
A. 4.67 m/s
B. 5.22 m/s
C. 2.57 m/s
D. 3.25 m/s

19. A sealed tank containing seawater to a height of 12 m also contains air above the water at gage
pressure of 5 atm. Water flows out from the bottom through a small hole. Calculate the efflux speed
of the water.
Note: ρs.w = 1.03 x 103 kg/m3. Answer: A
A. 34.92 m/s
B. 45.24 m/s
C. 20.45 m/s
D. 35.45 m/s

20. A cylindrical bucket, open at the top, 0.200 m high and 0.100 m in diameter. A circular hole with
cross- section area 1.0 cm2 is cut in the center of the bottom of the bucket. Water flows into the
bucket from a tube above it at the rate of 1.30 x 10 -4 m3/s. How high will the water in the bucket
rise? Answer: A
A. 8.62 cm
B. 4.67 cm
C. 2.52 cm
D. 1.25 cm

21. Modern airplane design calls for a lift due to the net force of the moving air on the wing of about
2000 N/m2 of wing area. Assume the air density (density = 1.2 kg/m 3) flows past the wing of an air-
craft with streamline flow. If the speed of flow put the lower wing surface is 140 m/s, what is the
required speed over the upper surface to give a lift of 2000 N/m 2? Answer: A
A. 151 m/s
B. 170 m/s
C. 172 m/s
D. 140 m/s

22. Water leaves a faucet with a downward velocity of 3 m/s. As the water falls below the faucet it
accelerates with accelerating g. the cross sectional area of the water stream leaving the faucet is 1.0
cm2. What is the cross- sectional area of the stream 0.50 m below the faucet? Answer: A
A. 0.69 cm2
B. 0.45 cm2
C. 0.57 cm2
D. 0.25 cm2

23. The density of ice is 917 kg/m3, and the approximate density of sea water in which an iceberg floats
is 1025 kg/m3. What fraction of iceberg is beneath the water surface? Answer: A
A. 0.89
B. 0.71
C. 0.45
D. 0.29

24. A hose shoots water straight up a distance of 2.5 m. The end opening on the hose has an area of
0.75 cm2. How much water comes out in min.? Answer: A
A. 31.5 liters
B. 41.5 liters
C. 20.5 liters
D. 130.5 liters

25. A barrel contains a 0.150 layer of oil floating on water that is 0.30 mm deep. If the density of the
oil is 600 kg/m3, what is the pressure (gage) at the oil-water interface? Answer: A
A. 883 Pa
B. 742 Pa
C. 900 Pa
D. 924 Pa

26. A hollow sphere is held below the surface of a fresh water lake by a cable anchored to the bottom
of the lake. The sphere has a volume of 0.300 m3, and the tension on the cable is 900 N. Calculate
the buoyant force exerted by the water on the sphere, and the mass of the sphere. Answer: A
A. 2,943 kN, 208.25 kg
B. 3.421 kN, 435.25 kg
C. 1,576 kN, 307.35 kg
D. 2,567 kN, 302.45 kg

27. What is the force acting on one side of the tank moving upward with an acceleration of 4.7 m/s 2
when it is filled with oil whose specific grsvity is 0.752 and height of 1.3 m for the bottom of the
tank. Answer: A
A. 11.986 kN
B. 15.723 kN
C. 14.572 kN
D. 17.457 kN

28. A vertical sliding gate is being submerged in a 10 meter water. The gate is 5 m wide and 9 m high
and weighs 50 kN. What vertical force is needed to just lift this gate considering that the coef. Of
friction between the guides and edges is 0.25. (Note: Neglect Buoyant Force ). Answer: C
A. 675 kN
B. 765 kN
C. 657 kN
D. 756 kN
29. A block of wood floats in water with 7 inches projecting above the water surface. It is then placed
in alcohol of sp. Gr. 0.82, and projected 4 in. above the surface. What is the sp. Gr. Of the block?
Answer: A
A. 0.66
B. 0.98
C. 0.68
D. 0.73

30. How far below the water surface should a vertical square 1 m on a side with two sides horizontal to
be immersed so that the center of pressure will be 70 mm below the center of gravity? Answer: A
A. 0.69 m
B. 0.72 m
C. 0.57 m
D. 0.78 m

31. A conical reducing sections connects an existent 10.16 cm diameter pipeline with a new 5.08 cm
diameter line. At 689.30 kPa static pressure under no-flow conditions, what is the tensile force is
exerted on the connectors at joint A assuming no end restraint from the pipe. Answer: A
A. 4.19 kN
B. 5.12 kN
C. 3.23 kN
D. 4.15 kN

32. An oil flowing at the rate of 0.001 m 3/s through an 8 cm diameter pipe has a kinematic viscosity of
2 x 10-5 m2/s and an specific gravity of 0.85. What is the centerline velocity? Answer: A
A. 0.398 m/s
B. 0.678 m/s
C. 0.235 m/s
D. 0.547 m/s

33. An ore sample weighs 15 N in air. When the ample is suspended by light chord and totally
immersed in water, the tension in the chord is 10.80 N. find the Total volume of the sample.
Answer: D
A. 2.54 x10-4 m3
B. 3.45 x10-4 m3
C. 5.67 x10-4 m3
D. 4.28 x10-4 m3

34. Water at 20 ºC is flowing in a pipe of radius 1.0 cm. The viscosity of water at 20 ºC is 1.005
centipoise. If the flow speed at the center is 0.200 m/s and the flow is laminar, find the pressure
drop de to viscosity along a 5 m section pipe. Answer: A
A. 40.2 Pa
B. 12.5 Pa
C. 13.45 Pa
D. 45.77 Pa

35. What is the specific gravity of a solid sphere that has a radius of 15 mm and a mass of 0.038 kg?
Answer: A
A. 2.687
B. 3.123
C. 2.157
D. 3.145

36. A piece of cork has a specific gravity of 0.25 and weighs 4 lb in air. If the weight density of water
is 62.4 lb/ft3. Compute the volume of the cork. Answer: C
A. 0.28 ft3
B. 0.24 ft3
C. 0.26 ft3
D. 0.38 ft3

37. At a height 10,000 m (33,000 ft) above the sea level, atmospheric pressure is about 210 mm of Hg.
What is the resultant normal force on a 600 cm 2 window of an airplane flying at this height when
the hydrostatic conditions and a pressure inside the plane of 760 mm of mercury and density of
mercury is 13,600 kg/m3? Answer: A
A. 4.403 kN
B. 3.407 kN
C. 5.721 kN
D. 2.741 kN

38. An atmospheric pressure of 101.325 kPa will be supported by how much height of water? Answer:
A
A. 10.32 m
B. 15.42 m
C. 13.23 m
D. 15.45 m

39. A small block of wood of density 0.4 x 10 3 kg/m3 is submerged in water at a depth of 2.9 m. Find
the acceleration of the block toward the surface when the block is released in a water whose
viscosity is to be negligible. Answer: A
A. 12.7 m/s2
B. 11.4 m/s2
C. 14.77 m/s2
D. 10.77 m/s2

40. The surface tension of water is 0.07 N/m. Find the weight of water supported by surface tension in
a capillary tube with radius of 0.10 mm. Answer: A
A. 44 µN
B. 54 µN
C. 35 µN
D. 57 µN

41. A submarine is 100 m long. The shape of its hull is roughly cylindrical with a diameter of 15 m.
When it is submerged, it cruises at a speed of about 40 knots or approximately 20 m/s. Compute the
reynold’s number, if the viscosity is 1.0 x 10-3 Pa.s. Answer: A
A. 3 x 108
B. 4 x 108+
C. 5 x 108
D. 2.5 x 108

42. The jet discharges water at the rate of 0.10 m 3/s and at a speed of 18 m/s to stationary plate.
Determine the force on the plate. Answer: A
A. 2800 N
B. 2000 N
C. 1800 N
D. 2500 N

43. A hydraulic lift is to be used to lift a truck weighing 5000 lb. What is the pressure (gage) must be
applied to the oil if the diameter of the large piston is 12 in. Answer: A
A. 44.2 psi
B. 42.7 psi
C. 14.5 psi
D. 17.6 psi

44. How high does mercury barometer stands on a day when atmospheric pressure is 98.6 kPa
considering that the specific gravity of mercury is 13.6. Answer: A
A. 739 mm
B. 400 mm
C. 760 mm
D. 540 mm

45. A rigid container is closed at one end and measures 8 in. diameter by 12 in. long The container is
held vertically and is slowly moved downward until the pressure in the container is 15.5 psia. What
will be the depth of the water surface measure from the free water surface? Answer: C
A. 9.2 in
B. 12 in
C. 22 in
D. 9.8 in

46. What is the expected head loss per mile of a closed circular pipe with 17 in. inside diameter friction
factor of 0.03 when 3300 gal/min of water flow under pressure? Answer: C
A. 0.007 ft
B. 0.647 ft
C. 38 ft
D. 35.80 ft
47. The fluid in a manometer tube is 60% water and 40% alcohol (S.G = 0.8). What is the manometer
fluid height difference if a 6.2 psi pressure is applied across the two ends of a manometer?
Answer: A
A. 215 in
B. 316 in
C. 151 in
D. 18.6 in

48. A 150 mm diameter pipe carries 81.3 L/s of water. The pipe branches into two pipes one is 50 mm
in diameter and the other is 75 mm. What is the velocity in 75 mm pipe if the velocity of the flow
in the 50 mm pipe is 10 m/s? Answer: A
A. 13.96 m/s
B. 15.67 m/s
C. 17.25 m/s
D. 19.43 m/s

49. A gas flows through a square conduit, at the entrance the conduit sides are 10 cm, the velocity is
7.55 m/s, and the gas mass density is 1.09 kg/m 3. At the exit, the conduit sides are 25 cm, and the
velocity of the flow is 2.02 m/s. Find the density of the gas at the exit section considering that the
gas is compressible. Answer: C
A. 0.6598 kg/m3
B. 0.6851 kg/m3
C. 0.6518 kg/m3
D. 0.6185 kg/m3

50. A hose shoots water straight up for a distance of 2.5 m. The end opening on the hose has an area of
0.75 cm2. How much water come out in 1 hour. Answer: A
A. 1.89 m3
B. 0.78 m3
C. 1.67 m3
D. 2.56 m3

INTERNAL COMBUSTION ENGINE


1. An OTTO Engine has a clearance volume of 7%. It produces 300 kW of power. What is the amount
of heat rejected in kW? Answer: C
A. 170
B. 160
C. 152
D. 145

2. An air standard has a compression ratio of 20 and a cut-off ratio of 5. If the air pressure and
temperature are 100 kPa and 27 °C, find the work in kJ/kg. Answer: B
A. 2796
B. 2165
C. 2437
D. 2751

3. Determine the air-standard efficiency of an engine operating on the Diesel cycle when the suction
pressure is 99.97 kPa and the fuel is injected for 6% of the stroke, the clearance volume is 8% of
the stroke. Assume k = 1.4. Answer: A
A. 60.07%
B. 65.01%
C. 67.01%
D. 64.02%

4. In an air standard OTTO cycle, the clearance volume is 18% of the displacement volume. Find the
thermal efficiency. Answer: C
A. 0.52
B. 0.55
C. 0.53
D. 0.60
5. In an air standard Diesel cycle, compression starts at 100 kPa and 300 K. the compression ratio is
16 to 1. The maximum cycle temperature is 2031 K. determine the thermal efficiency. Answer: A
A. 60.27%
B. 62.27%
C. 63.27%
D. 64.27%

6. A Diesel engine is operating on a 4-stroke cycle, has a heat rate of 11,315.6 kJ/kW-hr brake. The
compression ratio is 13. The cut-off ratio is 2. Using k = 1.32, what is the brake engine efficiency?
Answer: A
A. 63.5
B. 51.2
C. 73.5
D. 45.3

7. The compression ratio of an ideal OTTO cycle is 6:1. Initial conditions are 101.3 kPa and 20 °C.
Find the pressure and temperature at the end of adiabatic compression. Answer: A
A. 1244.5 kPa, 599.96 K
B. 1244.5 kPa, 60 °C
C. 1244.5 kPag, 60 °C
D. 1244.5 kPa, 599.96 °C
8. An engine operates on the air-standard OTTO cycle. The cycle work is 1000 kJ/kg. What is the
compression ratio of the engine if the maximum cycle temperature is 3173 K and the temperature
at the end of isentropic compression is 773 K. Answer: B
A. 6.85
B. 8.85
C. 7.85
D. 9.85

9. The conditions at the beginning of compression in an OTTO engine operating on hot-air standard
with k = 1.35 are 101.325 kPa, 0.05 m3 and 32 °C. The clearance is 8% and 15 kJ are added pr=er
cycle. Determine the mean effective pressure. Answer: A
A. 323.97 kPa
B. 423.97 kPa
C. 223.97 kPa
D. 523.97 kPa

10. The compression ratio of an OTTO cycle is 9. If the initial pressure is 150 kPa, determine the final
pressure. Answer: D
A. 1,251.10 kPa
B. 4,251.10 kPa
C. 2,251.10 kPa
D. 3,251.10 kPa

11. There are supplied 400 kJ per cycle to an ideal Diesel engine operating on 500 g air, P 1 = 100 kPa,
t1 = 50 °C. At the end of compression, P 2 = 4000 kPa. Determine the effective pressure of the
engine. Answer: C
A. 657.57 kPa
B. 457.57 kPa
C. 557.57 kPa
D. 357.57 kPa

12. What is the final temperature after compression of a diesel cycle if the initial temperature is 32 °C
and the clearance is 8%. Answer: A
A. 863.84 K
B. 763.84 K
C. 963.84 K
D. 663..84 K

13. An Ideal Dual Combustion cycle operates on 500 gram of air. At the beginning of the compression,
the air is at 100 kPa, 45 °C. If rp = 1.5, rc = 1.65, and rk = 10, determine the cycle efficiency.
Answer: D
A. 53.88%
B. 54.88%
C. 55.88%
D. 56.88%

14. The compression ratio of an engine working on the constant volume cycle is 9.3 to 1. At the
beginning of compression the the temperature is 31 °C and at the end of combustion the
temperature is 1205 °C. Taking the compression and expansion to be adiabatic and the value of k as
1.4. Calculate the theoretical thermal efficiency. Answer: A
A. 59.02%
B. 54.02%
C. 58.02%
D. 56.02%

15. The compression ratio in a Diesel engine is 13 to 1 and the ratio of expansion is 6.5 to 1. At the
beginning of compression the temperature is 32 °C. Assuming adiabatic compression and
expansion calculate the ideal thermal efficiency taking the specific heats at constant pressure and
volume as 1.005 kJ/kg-K and 0.718 kJ/kg-K respectively. Answer: C
A. 54%
B. 56%
C. 58%
D. 52%

16. The stroke of a gas engine which works on the constant volume cycle is 450 mm. The pressure at
the beginning of the compression is 1.01 bar and at the end of compression it is 11.1 bar. Assuming
compression follows the law PV1.36 = C, calculate the clearance between the piston and cylinder
cover at the end of compression in mm of length. Answer: C
A. 73.26 mm
B. 83.26 mm
C. 93.26 mm
D. 103.26 mm

17. The ratio of compression in a Diesel engine is 16 to 1 and the temperature of the air at the
beginning of the compression is 49 °C. Calculate the temperature at the end of compression
assuming it follows the law PV1.34 = C. Answer: B
A. 826.53 °C
B. 553.53 °C
C. 453.53 °C
D. 653.53 °C

18. The stroke of the piston in an internal combustion engine is 880 mm and the clearance is equal to
80 mm. The law of compression is PV1.38 = C and the pressure at the end of compression is 32 bar.
Find the increase of the final pressure caused by reducing clearance by 5 mm. Answer: C
A. 3.82 bar
B. 1.82 bar
C. 2.82 bar
D. 4.82 bar

19. In an internal combustion engine working on the constant volume cycle, the pressure, volume and
temperature at the beginning of compression are 101.325 kPa, 113 liters, 48°C respectively. During
combustion at constant volume the gas receives 95 kJ of heat. Taking compression according to the
law PV1.37 = C, specific heat at constant volume 0.712 kJ/kg-K and gas constant R = 00.29 kJ/kg-K,
calculate the mass of gas compressed. Answer: A
A. 0.123 kg
B. 0.245 kg
C. 0.346 kg
D. 0.456 kg

20. The pressure and temperature of the air at the beginning of compression in a diesel engine is 111.46
kPa and 35 °C respectively, and the clearance volume is equal to 7.5% of the piston swept volume.
Calculate the pressure at the end of compression assuming the law of compression if PV 1.36 = C.
Answer: C
A. 352.83 kPa
B. 8,082.90 kPa
C. 4,166.31 kPa
D. 41.11 kPa

21. The stroke of a petrol engine is 87.5 mm and the clearance is equal to 12.5 mm. A compression
plate is now fitted which has the effect of reducing the clearance to 10 mm. Assuming the
compression period to be the whole stroke, the pressure at the beginning of compression as 98.29
kPa and the law of compression PV 1..35 = C, calculate the pressure at the end of compression before
and after the compression plate is fitted. Answer: A
A. 1628.10 kPa
B. 1528.10 kPa
C. 1728.10 kPa
D. 1828.10 kPa

22. The compression ratio of a petrol engine working on the constant volume cycle is 8.5. The pressure
and temperature at the beginning of compression are 101.325 kPa and 40 °C and the maximum
pressure of the cycle is 3,141.08 kPa. Taking compression to follow the law PV 1.35 = C, calculate
the temperature at the end of compression. Answer: A
A. 388.97 °C
B. 288.97 °C
C. 355.65 °C
D. 255.65 °C

23. The stroke of an internal combustion engine is 75 mm, the diameter of the cylinder is 70 mm and
the clearance volume at the end of compression is 36 cm 3. Assuming compression follows the law
PV1..3 =C. Calculate the pressure at the end of compression if the initial pressure is 98.29 kPa.
Answer: B
A. 1 Mpa
B. 2 Mpa
C. 3 Mpa
D. 4 Mpa

24. A petrol engine working on the constant volume cycle has a compression ratio of 9 to 1. If the
pressure and temperature of the petrol-vapor air mixture at the beginning of compression are 99.30
kPa and 38 °C respectively. Calculate the temperature at the end of compression assuming it
follows the law PV1.24 = C. Answer: C
A. 13.45 °C
B. 28.45 °C
C. 99.30 °C
D. 83.48 °C

25. When the piston is moving up in a two-stroke diesel engine, the scavenge ports are closed when the
piston is 675 mm from the top of its stroke, the pressure and temperature of the air in the cylinder
then being 13.80 kPag and 43 °C. The clearance is equal to 65 mm and the diameter of the cylinder
is 650 mm. Calculate the mass of air compressed in the cylinder taking the atmospheric pressure as
101.325 kPa and R for air as 0.287 kJ/kg-K. Answer: B
A. 0.21 kg
B. 0.31 kg
C. 0.41 kg
D. 0.51 kg

26. The compression ratio of a compression-ignition engine is 14 to 1. The diameter of the cylinder is
500 mm and the stroke to bore ratio is 1.2 to 1. At the beginning of compression the pressure and
temperature of the air in the cylinder is 105 kPa and 51 °C, and compression follows the law PV n =
C, where n = 1.35. Calculate the work done on the air during compression. Answer: A
A. -57.69 kJ
B. -67.59 kJ
C. -76.59 kJ
D. -95.67 kJ

27. Gas initially at a pressure of 101.325 kPa and temperature of 60 °C undergoes the following cycle.
1-2 Adiabatic compression through compression ratio of 4.5:1.
2-3 Heating at constant volume through a pressure ratio of 1.35:1.
3-4 Constant pressure expansion to initial pressure.
4-1 Constant pressure cooling to initial volume.
If Cp = 1 kJ/kg-K and Cv = 0.678 kJ/kg-K for the gas, determine the thermal efficiency of the
cycle. Answer: A
A. 41.41%
B. 39.39%
C. 43.43%
D. 45.45%

28. One kilogram of air at a pressure and temperature of 101.325 kPa and 15 °C initially, undergoes the
following process in a cycle:
1-2 Isothermal compression to 202.65 kPa
2-3 Polytropic compression from 202.65 kPa to 405.30 kPa
3-1 Isentropic expansion from 405.30 kPa to initial condition
Calculate the heat transfer for process 1 to 2. Use R = 0.287 kJ/kg-K and k = 1.4 for air. Answer:
A
A. -57.29 kJ
B. -30.21 kJ
C. 70.30 kJ
D. 100.50 kJ

29. The compression ratio of an engine working on the dual combustion cycle is 10.7. The pressure and
temperature of the air at the beginning of compression is 101.325 kPa and 32 ºC. The maximum
pressure and temperature during the cycle is 4.154 Mpa and 1593 ºC. Assuming adiabatic
compression and expansion, calculate the temperature at the end of compression. Answer: B
A. 414.14 ºC
B. 514.14 ºC
C. 314.14 ºC
D. 614.14 ºC

30. In an Otto engine if the compression ratio is 5.5, what is the percent clearance? Answer: A
A. 22.22%
B. 31.11%
C. 19.64%
D. 25.34%

31. The efficiency of an Otto engine is 55% and the stroke volume is 35 liters. If the heat added at the
beginning of the combustion is 12.6 kJ, calculate the mean effective pressure. Answer: C
A. 194 kPa
B. 188 kPa
C. 198 kPa
D. 184 kPa

32. Gas is compressed in an internal combustion engine according to the PV 1.36 = C. if the initial and
final temperatures of the gas are 30 ºC and 382 ºC respectively, calculate the compression ratio.
Answer: D
A. 5.51
B. 6.51
C. 7.51
D. 8.51

33. If the volumes at the end of compression and at the beginning of expansion in an ideal Diesel
engine are 0.015 m3 and 0.040 m3 respectively. What is the cut-off ratio? Answer: B
A. 1.67
B. 2.67
C. 2.54
D. 1.54

34. The pressure ratio during the constant volume portion of combustion in a Dual Combustion engine
is 1.5. If the pressure at the end of the constant volume addition of heat is 4, what is the pressure at
the beginning? Answer: C
A. 3.67 Mpa
B. 6 Mpa
C. 2.67 Mpa
D. 7 Mpa

35. If the compression ratio and the cut-off ratio of a Diesel engine are 15 and 3 respectively, what is
the expansion ratio? Answer: B
A. 4
B. 5
C. 6
D. 7

36. At the beginning of compression in an ideal dual combustion cycle, the working fluid is 2 lbs of air
at 14.7 psia and 85 ºF. The compression ratio is 8.5, the pressure at the end of the constant volume
addition of heat is 450 psia, and there are added 105 Btu during the constant pressure expansion.
Find the pressure ratio and the percentage clearance? Answer: A
A. 1.53, 13.33%
B. 1.31, 12.42%
C. 1.53, 12.42%
D. 1.31, 13.33%

37. What is the efficiency of the Diesel Engine if the heat added during the constant pressure
combustion is 19 Btu and the heat rejected is 9 Btu. Answer: B
A. 42.63%
B. 52.63%
C. 55.66%
D. 45.66%

38. A Diesel cycle operates with a compression ratio of 14 and with a cu-off occurring at 5% of the
stroke. Find the cut-off ratio, if the initial volume is 1.5 ft 3. Answer: A
A. 1.64
B. 2.64
C. 3.64
D. 4.64

39. The air is compressed to 1/5 of its original volume in an Otto cycle. What is the final temperature
of the air if the initial temperature is 25 ºC. Answer: A
A. 288.58 ºC
B. 388.58 ºC
C. 255.66 ºC
D. 355.66 ºC

40. An Otto cycle operates 1 kg of air from 101.325 kPa and 25 ºC at the beginning of compression. If
the clearance is 15%, find the temperature at the end of compression. Answer: B
A. 300 ºC
B. 400 ºC
C. 450.01 ºC
D. 350.01ºC

41. The compression ratio and the cut-off ratio of an ideal diesel cycle are 13 and 3 respectively. What
is the cycle efficiency for a cold-air standard value of k. Answer: A
A. 53.20%
B. 43.20%
C. 55.21%
D. 45.21%

42. The efficiency of an ideal Otto cycle is 40%. What is the hot-air value of k, if the compression ratio
is 6.2? Answer: A
A. 1.28
B. 1.38
C. 1.25
D. 1.35

43. What is the clearance of an ideal Otto engine if the efficiency is 40% and k = 1.3? Answer: B
A. 20.24%
B. 22.27%
C. 10.24%
D. 12.27%

44. An Otto cycle operates a 0.5 lb/s of air from 15 psia and 150 ºF,at the beginning of compression.
The compression ratio is 4.3; hot air standard k = 1.3. Determine the volume at the end of
compression. Answer: B
A. 2.75 ft3/s
B. 1.75 ft3/s
C. 3.75 ft3/s
D. 4.75 ft3/s

45. An ideal dual combustion cycle operates on 0.60 kg of air. Find the cycle efficiency if the
compression ratio is 12, pressure ratio is 1.6, cut-off ratio 1.5 and k = 1.3. Answer: B
A. 40.71%
B. 50.71%
C. 60.71%
D. 45.71%

46. What is the pressure ratio in an ideal dual combustion cycle if the pressures at the beginning and
end of the constant volume portion of combustion are 2500 kPa and 4000 respectively. Answer: C
A. 1.4
B. 1.5
C. 1.6
D. 1.7

47. In an ideal dual combustion cycle if the cut-off ratio is 1.5 and the volume at the beginning of the
constant pressure portion of combustion is 0.055 m 3, what is the volume at the end of combustion?
Answer: A
A. 0.0625 m3
B. 0.0725 m3
C. 0.0825 m3
D. 0.055 m3

48. At the beginning of compression in an ideal dual combustion cycle, the working fluid is at 90 ºF. If
the pressure ratio and compression ratio are 1.6 and 13 respectively, what is the temperature at the
beginning of the constant pressure portion of combustion? Answer: C
A. 1886.64 ºF
B. 1495.05 ºF
C. 1995.05 ºF
D. 1586.64 ºF

49. An ideal dual combustion cycle operates on 0.65 kg of air. At the beginning of compression, the air
is at 100 kPa, 45 ºC. Determine the volume at the end of compression if net work is 300 kJ and the
mean effective pressure is 700 kPa. Answer: A
A. 0.16 m3
B. 0.26 m3
C. 0.12 m3
D. 0.22 m3

50. In an ideal dual combustion cycle, what is the pressure at the beginning of constant pressure
combustion if the pressure rario is 1.4 and the pressure at the beginning of the constant volume
portion of combustion is 2500 kPa. Answer: B
A. 3000 psi
B. 3500 psi
C. 3200 psi
D. 3600 psi

51. In an air-standard Otto cycle, air enters at 104.7 kPa and 27 ºC. The compression rario is 7:1. If the
net work of the cycle is 120 kJ/kg. Calculate the heat supplied in kJ/kg? Answer: A
A. 221.88 kJ/kg
B. 122.88 kJ/kg
C. 188.22 kJ/kg
D. 288.12 kJ/kg

52. A diesel engine takes in air at 105 kPa and 26 ºC subjected to a compression ratio of 19. What is
the operating clearance in percent? Answer: A
A. 5.56%
B. 6.66%
C. 4.56%
D. 7.56%
53. The volumes of states 1,2 and 3 of a diesel cycle operating on a hot air standard are 300 cm 3, 30
cm3 and 90 cm3 respectively. Find the power output if the heat input is 120 kW. Answer: A
A. 57.63 kW
B. 77.63 kW
C. 87.63 kW
D. 97.63 kW

54. An ideal diesel engine operates on a 1 kg of cold air with a suction pressure of 110 kPa and an
initial temperature of 40 ºC. The compression pressure is 3000 kPa and the cut-off is at 8% of the
stroke from head-end center of the piston. Calculate the ideal cycle thermal efficiency. Answer: C
A. 58.56%
B. 59.55%
C. 55.85%
D. 65.56%

55. The air-standard dual cycle starts at 105 kPa and 26 ºC. The compression ratio is 11:1 and the total
heat intake is 300 kJ/kg. If the heat intake at constant volume is 3/5 of the total, determine the mean
effective pressure in kPa. Answer: A
A. 247.56 kPa
B. 517.67 kPa
C. 347.34 kPa
D. 457.34 kPa

FUELS AND COMBUSTION


1. A bituminous coal has the following compositions:
C =71.5 % O = 7.0% S = 3.6% W = 3.4%
H = 5.0% N = 1.3% Ash = 8.2%
Calculate for complete combustion the theoretical weight of air required in kg air/kgcoal. Answer: A
A. 9.80
B. 8.80
C. 7.80
D. 6.80

2. Calculate the higher heating value of liquid dodecane fuel. The chemical formula of Dodecane is
C12H26. Answer: A
A. 48,255 kJ/kg
B. 55,641 kJ/kg
C. 60,221 kJ/kg
D. 35,245 kJ/kg

3. Calculate the minimum volume of day tank of 28 °API fuel having a fuel consumption of 1 kg/s.
Answer: B
A. 94.71 m3
B. 97.41 m3
C. 91.74 m3
D. 79.41 m3

4. It is required to find the theoretical volume of air at 20 °C and 100 kPa absolute pressure to burn
one kilogram of Franklin Country coal. The ultimate analysis of coal-as-fired is as follows:
C = 65.65% O = 18.61% Mois. = 3%
H = 5.87% S = 1.51% Ash = 5.36%
Answer: B
A. 9.43 m3air/kgcoal
B. 7.43 m3air/kgcoal
C. 8.43 m3air/kgcoal
D. 6.43 m3air/kgcoal

5. A 650 BHP diesel engine uses fuel oil of 28 °API gravity, fuel consumption is0.65 lb/BHP-hr. cost
of fuel is P 7.95 per liter. For continuous operation, determine the minimum volume of cubical day
tank in cm3, ambient temperature is 45 °C. Answer: C
A. 4,372,890 cm3
B. 5,987,909 cm3
C. 5,291,880 cm3
D. 7,352,789 cm3

6. A logging firm in Isabela operates a Diesel Electric plant to supply its electric energy requirements.
During a 24 hr period, the plant consumed 250 gallons of fuel at 80 °F and produced 2700 kw-hrs.
Industrial fuel used is 30 °API and was purchased at P 3.00 per liter at 60 °F. Determine the over-
all efficiency of the plant. Answer: A
A. 26.08%
B. 25.06%
C. 29.07%
D. 30.01%

7. A circular tank 45 ft long and 5.5 ft diameter is used for oil storage. Calculate the number of days
the supply tank can hold for continuous operation at the following conditions.
Steam flow = 200lb/hr
Steam dry and saturated at 200 psia
Feedwater temperature = 230 °F
Boiler efficiency = 75%
Fuel oil = 34 °API
Answer: A
A. 15.21 days
B. 16.23 days
C. 17.57 days
D. 19.26 days

8. Liquid Octane (C8H18) fuel is burned with ideal proportion of air. Calculate the ideal air-fuel ratio
by weight. Answer: C
A. 13.72 kgair/kgfuel
B. 14.56 kgair/kgfuel
C. 15.05 kgair/kgfuel
D. 10.69 kgair/kgfuel
9. A typical industrial fuel oil, C16H32 with 20% excess air, by weight. Assuming complete oxidation
of the fuel, calculate the actual air-fuel ratio by weight. Answer: A
A. 17.56 kgair/kgfuel
B. 15.76 kgair/kgfuel
C. 16.75 kgair/kgfuel
D. 17.65 kgair/kgfuel

10. The ultimate analysis of coal is given below:


C = 68.5% S = 1.5% Ash = 12.0%
H = 2.5% 3.5%
Calculate the higher heating value of coal. Answer: D
A. 25,300 kJ/kg
B. 28,300 kJ.kg
C. 27,300 kJ/kg
D. 26,300 kJ/kg

11. Fuel oil in a day tank for use of an industrial boiler is tested with hydrometer. The hydrometer
reading indicates a S.G = 0.924 when the temperature of the oil in the tank is 35 °C. Calculate the
higher heating value of the fuel. Answer: A
A. 43,852.13 kJ/kg
B. 53,852.13 kJ/kg
C. 58,352.13 kJ/kg
D. 48,352.13 kJ/kg

12. A steam generators burns fuel oil with 20% excess air. The fuel oil may be represented by C 14H30.
The flue gas leaves the preheater at 0.31 Mpa. Find the minimum stack temperature to avoid
condensation. Answer: A
A. 73 °C
B. 63 °C
C. 83 °C
D. 53 °C

13. Gaseous fuel mixture has a molal analysis:


H2 = 14% CH4 = 3% CO = 27%
O2 = 0.6% CO2 = 4.35% N2 = 50.9%
Determine the air-fuel ratio for complete combustion on molal basis. Answer: A
A. 2.130
B. 3.230
C. 1.233
D. 1.130

14. In a boiler design, it is desirable to have the flue gas exit temperature above the dew point.
Estimate the dew point temperature of the flue gas produced by combustion having the gravimetric
analysis of:
N2 = 71.84% O2 = 3.61%
CO2 = 20.35% H2O = 4.20%
Assume that air infiltration and leakage are negligible. Answer: A
A. 39 ºC
B. 41 ºC
C. 42 ºC
D. 43 ºC

15. If the theoretical air-fuel ratio is 15, what is the approximate higher heating value in kJ/kg fuel?
Answer: A
A. 46,755 kJ/kg
B. 56,755 kJ/kg
C. 45,765 kJ/kg
D. 55,765 kJ/kg

16. There are 20 kg of flue gases formed per kg of fuel oil burned in the combustion of a fuel oil
C12H26. What is the excess air in percent? Answer: A
A. 20.17
B. 16.56
C. 26.67
D. 8.21

17. A diesel electric plant supplies energy for Meralco. During a 24 hr period, the plant consumed 200
gallons of fuel at 28 ºC and produced 3930 kW-hr. Industrial fuel used is 28 ºAPI and was
purchased at P 5.50 per liter at 15.6 ºC. What should the cost of fuel be produce one kW-hr?
Answer: A
A. P 1.05
B. P 1.10
C. P 1.069
D. P 1.00

18. A fuel oil is burned with 50% excess air. What is the volume rate of flow in m 3/min of the wet
products at a pressure of 102 kPa and a temperature of 350 ºC when the fuel is burned at the rate of
45 kg/min? Assume that the combustion requirements of the fuel oil are similar to those of C 12H26.
Answer: A
A. 1865
B. 3526
C. 2462
D. 4563

19. A flue gas has the following volumetric analysis:


CH4 = 68% C2H6 = 32%
Assume complete combustion with 15% excess air at 101.325 kPa, 21 ºC wet bulb and 27 ºC dry
bulb. What is the partial pressure of the water vapor in kPa? Answer: D
A. 9.62
B. 12.81
C. 17.28
D. 15.94

20. The following is an analysis of coal in percent:


C = 74 O2 = 8 S=1
H2 = 6 N2 = 1.6 Ash = 9.4
If burned in a boiler, the coal produces the following Orsat analysis in percent:
CO2 = 12 O2 = 6.5
CO = 0.1
The refuse contains 0.008 kg of carbon per kg of coal burned. Determine the percentage of excess
air used. Answer: A
A. 45.6%
B. 46.5%
C. 54.6%
D. 56.4%

21. A gaseous mixture has a dew point temperature of 15 ºC. the total pressure is 143.27 kPa.
Determine the amount of water vapor present in 100 moles of the mixture.
Note: saturation pressure at 15 ºC is 1.7051 kPa. Answer: A
A. 1.10
B. 2.19
C. 1.19
D. 2.0

22. A coal fired utility boiler has the following characteristics:


Coal Freed Rate _ _ _ _ _ _ _ _ _ _ _ 6,940 kg/hr
Electric Power Rating _ _ _ _ _ _ _ _ 50 MW
Refuse Removal Rate _ _ _ _ _ _ _ _ 410 kg/hr
Carbon in Refuse_ _ _ _ _ _ _ _ _ _ 30%
Ultimate Analysis in percent:
C = 76.56 O2 = 7.70 S = 2.44
H2 = 5.50 N2 = 1.70 Si = 6.10
Heating value of coal is 32,773 kJ/kg
If stack gas particle collectors are required to meet the limit of 0.043 kg particulates per million kg
of fuel feed set by Environmental management Bureau under the jurisdiction of the DENR, what
efficiency must the collection system have? Molecular weight of S i = 28.09. Answer: A
A. 99.999953%
B. 89.999953%
C. 79.999953%
D. 69.999953%

23. The dry exhaust gas from oil engine has the following gravimetric analysis:
CO2 = 21.6% N2 = 74.2%
O2 = 4.2%
Specific heats at constant pressure for each component of the exhaust gas in kCal/kgºC are:
CO2 = 0.203 N2 = 0.248
O2 = 0.219
Calculate the specific gravity if the molecular weight of air is 28.97 kg/kg mol. Answer: A
A. 1.055
B. 1.155
C. 1.255
D. 0.958

24. A coal fired steam boiler uses 3000 kg of coal per hour. Air required for combustion is 15.5 kg per
coal at a barometric pressure of 98.2 kPaa. The flue gas has a temperature of 285 °C and an average
molecular weight of 30. Assuming an ash loss of 11% and an allowable gas velocity of 7.5 m/s,
find the diameter of the chimney. Answer: A
A. 1.88 m
B. 1.64 m
C. 2.88 m
D. 2.64 m

25. A certain coal has the following ultimate analysis:


C = 70.5% H = 4.5% O2 = 6% N2 = 1.0%
S = 3.0% Ash = 11% Moisture = 4%
A stoker fired boiler of 175,000 kg/hr steaming capacity uses this coal as fuel. Calculate the
volume of air in m3/hr with air at 60 °F (15.6 °C) and 14.7 psia (101.325 kPa) the coal is burned
with 30% excess air. Boiler efficiency of 70% and factor of evaporation of 1.10. Answer: A
A. 212,861.04 m3/hr
B. 221,861.04 m3/hr
C. 218,261.04 m3/hr
D. 281,261.04 m3/hr

26. Methane gas burn completely when supplied with 30 percent excess dry air. The gaseous products
are cooled at 32 °C and 1 atm pressure. Determine the mass of moisture condensed per kg fuel.
Answer: D
A. 1.302
B. 1.924
C. 1.816
D. 1.614

27. A diesel power plant consumed 1 m3 of fuel with 30 °API at 27 °C in 24 hrs. Calculate the fuel rate
in kg/hr. Answer: A
A. 36.21
B. 26.25
C. 29.34
D. 39.42

28. A coal has the following ultimate analysis:


C = 60% H2 = 6% Ash = 5% S2 = 7%
N2 = 5% O2 = 9% Moisture = 8%
Determine the actual air-fuel ratio using 20% excess air. Answer: A
A. 10.66 kgair/kgcoal
B. 11.66 kgair/kgcoal
C. 12.66 kgair/kgcoal
D. 9.66 kgair/kgcoal
29. The ultimate analysis of a petroleum fuel is 87.1% carbon and 12.9% hydrogen. A certain test of
dry products of combustion gives the following analysis by volume: 12% CO 2; 0.30% CO; 4.6%
O2; 83.1% N2. Calculate the number of atoms of carbon and hydrogen of the hydrocarbon fuel.
Answer: A
A. n = 7.26, m = 12.90
B. n = 8.26, m = 7.26
C. n = 8.26, m = 13.9
D. n = 13.90, m = 8.26

30. The analysis of natural gas fuel used in a gas turbine power plant has the following percentages by
volume:
CH4 = 80% C2H4 = 5%
H2 = 10% CO = 2%
and the remaining non-combustible gases. Find the volume of air supplied per hour if the gas
consumption is 20,000 m3/hr assuming 40% excess air. Answer: A
A. 248,500 m3/hr
B. 348,500 m3/hr
C. 260,000 m3/hr
D. 360,000 m3/hr

31. A diesel power plant uses fuel with heating value of 43,000 kJ/kg. What is the density of the fuel at
25 °C? Answer: C
A. 840 kg/m3
B. 873 kg/m3
C. 970 kg/m3
D. 940 kg/m3

32. Calculate the calorific value of coal which has the following analysis: Carbon 81%, Hydrogen 5%,
Oxygen 5.6%, Sulfur 1% and the remainder is ash content. Answer: A
A. 33,688.35 kJ/kg
B. 32,135.45 kJ/kg
C. 12,456.34 kJ/kg
D. 32,457.45 kJ/kg

33. What is the calorific value and the minimum mass of air required per kilogram of fuel oil which is
composed of 85.2% carbon, 12% hydrogen, 1.6% oxygen and 1.2% impurities. Answer: A
A. 13.87 kgair/kgcoal
B. 14.55 kgair/kgcoal
C. 12.57 kgair/kgcoal
D. 15.75 kgair/kgcoal

34. A petrol has the following analysis 85.5% carbon, 14.4% hydrogen, and 0.10% sulfur. Calculate the
volume of air at 1.0 bar and 15 °C required for perfect combustion of 1 kg of the fuel. Note: R air =
0.287 kJ/kg-K. Answer: A
A. 12.24 m3/ kgfuel
B. 13.45 m3/ kgfuel
C. 15.54 m3/ kgfuel
D. 14.57 m3/ kgfuel

35. A fuel used in a boiler contains 86.1% carbon, 12.5% hydrogen, and 0.4% oxygen and 1% sulfur.
There are 40% excess air supplied to the furnace and the fuel rate is 400 kg/hr. Calculate the mass
of air and the heat energy transferred to the air per hour if it enters the air heater at 18 °C and
leaves at 130 °C. Note: Cp = 1.00kJ/kg-K. Answer: A
A. 7,974.26 kJ/kg, 890,093.12 kJ/kg
B. 8,678.45 kJ/kg, 980,045.12 kJ/kg
C. 7,345.35 kJ/kg, 670,035.45 kJ/kg
D. 8,987.76 kJ/kg, 567,098.57 kJ/kg

36. A fuel has the following constituents: Carbon 85%, Hydrogen 13%,Oxygen 2%. When burning this
fuel in a boiler furnace the air supply is 50% in excess of the theoretical minimum required for
complete combustion, the inlet temperature of the air being 31 °C and funnel temperature is 280
°C. Compute its calorific value, air-fuel ratio and percentage of heat energy supplied. Answer: A
A. 47,133.03 kJ/kg, 21.26 kgair/kgfuel
B. 35,897.54 kJ/kg, 12.21 kgair/kgfuel
C. 14,576.24 kJ/kg, 17.27 kgair/kgfuel
D. 45,457.13 kJ/kg, 12.54 kgair/kgfuel

37. Experiment revealed that the calorific value of an oil fuel by means of a bomb calorimeter, the
mass of a sample fuel was 0.75 gram, the mass of water surrounding the bomb was 1.8 kg with
some water in the fitting 0.47 kg, and the temperature rises by 3.3 °C. Considering that the specific
heat of water is 4.2 kJ/kg-K, compute the calorific value of this oil in Mj/kg. Answer: A
A. 41,949.6 kJ/kg
B. 35,345.2 kJ/kg
C. 47,457.23 kJ/kg
D. 31,687.45 kJ/kg

38. A fuel consists of 84% carbon, 13% hydrogen, and the remainder incombustible solid matter.
Calculate the calorific value of the theoretical mass air required per kg of fuel and the mass
analysis of the flue gas if 22 kg of air are supplied per kg of fuel burned. Answer: A
A. 46,795.83 kJ/kg, 14.06 kgair/kgfuel
B. 45,787.87 kJ/kg, 15.06 kgair/kgfuel
C. 34,576.78 kJ/kg, 14.75 kgair/kgfuel
D. 45,567.78 kJ/kg, 15.67 kgair/kgfuel

39. A fuel oil has the following contents: 85.35% carbon, 11.9% hydrogen, 1.6% oxygen, and 1%
impurities. Calculate the percentage CO2 in the flue gas for complete combustion. Answer: A
A. 20.94%
B. 30.94%
C. 35.5%
D. 17.56%

DIESEL POWER PLANT


1. A six cylinder, four stroke diesel engine with 76 mm bore x 89 stroke was run in the laboratory at
2000 rpm, when it was found that the engine torque was 153.5 N-m when one cylinder was out.
The engine consumed 12.2 kg of fuel per hour with a heating value of 54,120 kJ/kg of air at 15.6
°C. Determine the indicated power. Answer: B
A. 32.1 kW
B. 38.4 kW
C. 23.3 kW
D. 48.3 kW
2. A single-acting, four cylinder, 4 stroke cycle diesel engine with a bore to stroke of 21.59 cm x
27.94 cm, operating at 275 rpm, consumes 8.189 kg/hr of fuel whose heating value is 43,961.4
kJ/kg. The load on the brake arm, which is 93.98 cm is 113.4 kg. What is the brake arm mean
effective pressure? Answer: C
A. 412.20 kPa
B. 124.17 kPa
C. 319.95 kPa
D. 645.53 kPa

3. A supercharged six cylinder, four stroke cycle diesel engine of 10.48 cm bore and 12.7 cm stroke
has a compression ratio of 15. When it is tested on a dynamometer with a 53.34 cm arm at 2500
rpm, the scale reads 81.65 kg, 2.86 kg of fuel of 45,822.20 kJ/kg heating value are burned during a
6 min. test and air metered to the cylinders at the rate of 0.182 kg/s. Find the brake thermal
efficiency. Answer: C
A. 0.327
B. 0.367
C. 0.307
D. 0.357

4. In a test laboratory, it was found out that of the 80 Bhp developed by an engine test, 45 Hp are
absorbed by the cooling water that is pumped through the water jacket and the radiator. The water
enters the top of the radiator at 200 °F. At that temperature, enthalpy of the water is 168.307
Btu/lbm. Water leaves the bottom of the radiator at 190 °F and with an enthalpy of 158.03 Btu/lb m.
What is the water flow rate for a steady- state condition? Answer: C
A. 25 gal/min
B. 20 gal/min
C. 23 gal/min
D. 24 gal/min

5. A certain diesel engine with the following specifications, 8 cylinder, 400 mm x 600 mm, four
stroke cycle has a fuel consumption of 0.60 lbs/hp-hr based on 19,100 Btu/lb. Engine speed is 280
rpm with an indicated mean effective pressure of 130 psi. If the water jacket carries away an
estimated 25% of the heat supplied, find its capacity (GPM) required if the allowable rise is 40 °F.
Answer: A
A. 241.9 GPM
B. 236.5 GPM
C. 249.1 GPM
D. 268.7 GPM

6. A 2000 kW diesel engine unit uses 1 bbl oil per 525 kw-hr produced. Oil is 25 °API. Efficiency of
generator is 93%, mechanical efficiency of engine is 80%. What is the thermal efficiency of the
engine based on indicated power? Answer: A
A. 31.69%
B. 29.47%
C. 39.6%
D. 35.6%
7. Find the power which a 2.5 MW natural gas engine can developed at an altitude of 1981.2 m
taking into consideration that the pressure change alone. Answer: A
A. 1.957 MW
B. 1.597 MW
C. 1.795 MW
D. 1.579 MW

8. A 373 kW (500 Hp) internal combustion engine has a brake mean effective pressure of 551.5 kPa at
full load. What is the friction power if mechanical efficiency is 85%? Answer: A
A. 88.23 Hp
B. 77.23 Hp
C. 99.24 Hp
D. 66.24 Hp

9. A dynamometer test was done for one hour at steady load on a 6 cylinder diesel engine. It was
found to use 42 kg of fuel having Qh = 42,000 J/g. Cylinder is 22.8 cm x 27 cm, 4 cycle type.
Speed, 550 rpm and dynamometer torque at 27,000 kg-cm. Determine the brake thermal
efficiency. Answer: A
A. 31.15%
B. 29.16%
C. 32.36%
D. 35.52%

10. A four-stroke,8 cylinder diesel engine with bore and stroke of 9 inches and 12 inches respectively
and speed of 950 rpm has a brake mean effective pressure of 164 psi. The specific fuel
consumption is 0.39 lb/bhp-hr and the fuel heating value is 18,500 Btu/lb. Determine the thermal
efficiency. Answer: B
A. 32.57%
B. 35.27%
C. 57.35%
D. 45.57%

11. A four-stroke 394 mm bore and 534 mm stroke single acting diesel engine with four cylinder is
guaranteed to deliver 350 BHp at 300 rpm. The engine consumed 66.8 kg/hr of fuel with a heating
value of 44,251 kJ/kg. Calculate the indicated mean effective pressure in kPa if mechanical
efficiency is 89%. Answer: A
A. 451 kPa
B. 475 kPa
C. 490 kPa
D. 425 kPa

12. A 305 mm x 457 mm four stroke single acting diesel engine is rated at 150 kW at 260 rpm. Fuel
consumption at rated load is 0.26 kg/kW-hr with a heating value of 43,912 kJ/kg. Calculate the
brake thermal efficiency. Answer: A
A. 31.63%
B. 41.63%
C. 21.63%
D. 35.63%
13. A four cylinder, 4 stroke cycle, 20 cm x 25 cm x 550 rpm diesel engine has a mean effective
pressure of 150 psi. Calculate the brake power in kW if engine efficiency is 88%. Answer: A
A. 131 kW
B. 138 kW
C. 141 kW
D. 148 kW

14. A four stroke, direct injection diesel engine of 4 in. bore x 4.5 in. stroke develops 45 Hp at full load
and 8 Hp when running at idling load. Engine speed is 2,200 rpm. What is the mechanical
efficiency? Answer: A
A. 85%
B. 88%
C. 77%
D. 74%

15. A 500 kW diesel has a heat rate of 12,000 kJ/kw-hr. the compression ratio is 16:1, cut-off ratio of
2.3. Assume k = 1.32. Calculate the engine efficiency based on the output of 500 kW. Answer: A
A. 57.77%
B. 57.78%
C. 47.77%
D. 37.67%

16. If the mean effective pressure of a diesel engine is 500 kPa and the mechanical efficiency is 85%,
what is the indicated mean effective pressure? Answer: A
A. 588.24 kPa
B. 524.88 kPa
C. 688.24 kPa
D. 624.88 kPa

17. The brake thermal efficiency of a 1 MW diesel electric plant is 36%. Find the heat generated by
fuel in kW if the generator efficiency is 89%. Answer: A
A. 3,121.10 kW
B. 3,528.64 kW
C. 4,121.10 kW
D. 4,528.64 kW

18. A 500 kW diesel engine develops torque of 2 kN-m, The engine drives an alternating current
generator with 8 poles producing current 60 Hz. What is the speed ratio of speed reduction gear?
Answer: A
A. 2.6:1
B. 3.6:1
C. 1.6:1
D. 4.6:1

19. A diesel engine is operating on a 4 stroke cycle has a heat rate of 8000 Btu/Hp-hr. the compression
ratio is 13, cut-off ratio is 2 and k = 1.32. Find the brake engine efficiency. Answer: A
A. 63.49%
B. 64.39%
C. 69.43%
D. 46.93%
20. What is the mechanical efficiency of a 0.5 MW diesel engine if the friction power is 70 kW.
Answer: A
A. 87.72%
B. 84.55%
C. 89.61%
D. 88.24%

21. Gas is expanded in an engine cylinder following the law PV n = C where the value of n is 1.3. The
initial pressure is 2,250 kPa and the final pressure is 210 kPa. If the volume at the end of expansion
is 0.75 m3, calculate the volume at the beginning of expansion. Answer: A
A. 0.1099 m3
B. 0.099 m3
C. 0.9019 m3
D. 0.9910 m3

GAS TURBINE PLANT


1. A gas turbine working on air standard Brayton cycle has air enter into the compressor at
atmospheric condition and 22 °C. The pressure ratio is 9 and the maximum temperature in the cycle
is 1077 °C. Compute for the cycle efficiency per kg of air in percent. Answer: A
A. 44.85%
B. 43.92%
C. 41.65%
D. 42.62%

2. In a gas turbine unit, air enters the combustion chamber at 550 kPa, 227 °C and 43 m/s. The
products of combustion leave the combustor at 511 kPa, 1004 °C nd 140 m/s. Liquid fuel enters
with heating value of 43,000 kJ/kg. For fuel-air ratio of 0.0229, what is the combustor efficiency of
the unit in percent? Answer: C
A. 64%
B. 92%
C. 78%
D. 102%

3. Air enters the combustion chamber of a gas turbine unit at 550 kPa, 227 °C and 43 m/s. The
products of combustion leave the combustor at 517 kPa, 1007 °C nd 140 m/s. Liquid fuel enters
with heating value of 43,000 kJ/kg. The combustor efficiency is 95%. What is the air-fuel ratio?
Note: Properties of air:
T(K) h(kJ/kg)
500 503.02
1280 1372.25. Answer: D
A. 47.39
B. 32.25
C. 56.93
D. 44.95

4. Air enters the compressor of a gas turbine at 100 kPa and 300 K with a volume flow rate of 5 m 3/s.
The compressor pressure ratio is 10 and its isentropic efficiency is 85%. At the inlet to the turbine,
the pressure is 950 kPa and the temperature is 1400 K. The turbine has an isentropic efficiency of
88% and the exit pressure is 100 kPa. On the basis of an air-standard analysis, what is the thermal
efficiency of the cycle in percent? Answer: C
A. 42.06
B. 60.20
C. 31.89
D. 25.15

5. What is the thermal efficiency of an air-standard Brayton cycle if the pressure ratio is 10? Answer:
A
A. 48.21%
B. 50.16%
C. 45.36%
D. 42.44%

6. In a simple gas turbine plant working on the ideal constant pressure cycle, air is taken into the
compressor at 1 bar, 16 °C and delivered at 5.4 bar. If the temperature at turbine inlet is 700 °C.
Calculate the ideal thermal efficiency. Take k = 1.4. Answer: A
A. 38.23%
B. 42.61%
C. 45.66%
D. 35.38%

7. Air is drawn into a gas turbine working on the constant pressure cycle at 1 bar 21 °C and
compressed to 5.77 bar. The temperature at the end of heat supply is 680 °C. Taking expansion and
compression to be adiabatic where C v = 0.718 kJ/kg-K, Cp = 10.55 kJ/kg-K, calculate the heat
energy supplied per kg at constant pressure. Answer: A
A. 472 kJ/kg
B. 389 kJ/kg
C. 501 kJ/kg
D. 489 kJ/kg

8. What is the thermal efficiency of an air-standard Brayton cycle if the air enters and leaves the
turbine at 1000 K and 550 K respectively? Answer: B
A. 40.74%
B. 44.99%
C. 50.38%
D. 54.86%

9. There are required 2200 kW net from a gas turbine unit for pumping of crude oil. Air enters the
compressor section at 100 kPa, 280 K, the pressure ratio r p = 10. The turbine sections receives the
hot gases at 1,100 K. Assume the closed Brayton cycle and determine the required air flow.
Answer: C
A. 7.91 kg/s
B. 7.16 kg/s
C. 8.11 kg/s
D. 8.91 kg/s

10. In an air-standard Brayton cycle, the compressor receives air at 101.325 kPa, 21 °C and it leaves at
600 kPa at the rate of 4 kg/s. Determine the turbine work if the temperature of the air entering the
turbine is 1000 °C. Answer: C
A. 3000 kW
B. 2701 kW
C. 2028 kW
D. 3500 kW

11. The net power output of an air-standard Brayton cycle is 200 kW. Air enters the compressor at 32
°C and leaves the high-temperature heat exchanger at 800 °C. What is the mass flow rate of air if it
leaves the turbine at 350 °C? Answer: D
A. 0.57 kg/s
B. 0.67 kg/s
C. 0.77 kg/s
D. 0.87 kg/s

12. The compressor inlet air temperature in a gas turbine plant is 99 °C. Calculate the compressor air
exit temperature if it requires 400 kJ/kg of work. Answer: A
A. 499 °C
B. 400 °C
C. 500 °C
D. 599 °C

13. In an air-standard Brayton cycle air enters the compressor at 101.325 kPa and 27 °C. Determine the
net work if the maximum temperature is 1000 °C and the pressure ratio is 9. Answer: C
A. 456.88 kJ/kg
B. 421.56 kJ/kg
C. 331.47 kJ/kg
D. 301.74 kJ/kg

14. What is the efficiency of the compressor in a gas turbine plant if the compressor power is 300 kW.
Power input is 400 kW. Answer: A
A. 75%
B. 80%
C. 85%
D. 70%

15. The intake of the compressor of an air-standard Brayton cycle is 35,000 ft 3/min at 14 psia and 95
°F. The compression ratio is 4 and the temperature at the turbine inlet is 1500 °F. The exit pressure
of the turbine is 14 psia. Determine the mean effective pressure. Answer: A
A. 25.06 psi
B. 28.05psi
C. 35.06 psi
D. 38.05 psi

16. Calculate the the work done per kg of gas expanding from 6.33 kg abs to 1.05 kg/cm 2 abs. in a gas
turbine of 82% internal efficiency. Initial temperature, 750 °C; k = 1.34, M = 29. Answer: A
A. 349 kJ/kg
B. 425 kJ/kg
C. 249 kJ/kg
D. 525 kJ/kg

17. Kerosene is the fuel of a gas turbine plant: fuel-air ratio, m f = 0.012, T3 = 972 K, pressure ratio, rp =
4.5, exhaust to atmosphere. Find the available energy in KJ per kg air flow. Assume k = 1.34 and
Cp = 1.13. Answer: A
A. 352.64 kJ/kg
B. 452.64 kJ/kg
C. 252.64 kJ/kg
D. 552.64 kJ/kg

18. An ideal gas turbine operates with a pressure ratio of 10 and the energy input in the high
temperature heat exchanger is 300 kW. Calculate the air flow rate for a temperature limits of 30 °C
and 1200 °C. Answer: B
A. 0.25 kg/s
B. 0.34 kg/s
C. 0.41 kg/s
D. 0.51 kg/s
19. Products of combustion with k of 1.35, 556 K, molecular weight M = 29, are moving within an
exhaust pipe at 174 m/s; 1.12 kg/cm 2 abs. static pressure. Find the total pressure and temperature.
Answer: C
A. 1.61 kg/cm2, 580 K
B. 1.61 kg/m2, 570 K
C. 1.13 kg/cm2, 570 K
D. 1.13 kg/cm2, 580 K

20. In an air –standard Brayton cycle the inlet temperature and pressures are 20 ºC and 101.325 kPa.
The turbine inlet conditions are 1200 kPa and 900 ºC. Determine the air flow rate if the turbine
produces 12 MW. Answer: B
A. 21.41 kg/s
B. 20.20 kg/s
C. 19.25 kg/s
D. 18.10 kg/s

21. A gas turbine power plant operating on the Brayton cycle delivers 15 MW to a standby electric
generator. What is the mass flow rate and the volume flow rate of air if the minimum and
maximum pressures are 100 kPa and 500 kPa respectively, and temperatures of 20 ºC and 1000 ºC.
Answer: A
A. 31.97 kg/s, 26.88 m3/s
B. 36.98 kg/s, 28.99 m3/s
C. 41.97 kg/s, 26.88 m3/s
D. 46.98 kg/s, 28.99 m3/s

STEAM POWER PLANT


1. In a Rankine cycle, steam enters the turbine at 2.5 MPa and condenser of 50 kPa. What is the
thermal efficiency of the cycle?
Steam Properties: at 2.5 MPa, h g = 2803.1 kJ/kg, Sg = 6.2575 kJ/kg-K and at 50 kPa, S f = 1.0910
kJ/kg-K, hf = 340.49 kJ/kg, hfg = 2305.4 kJ/kg, vf = 0.0010300 m3/kg. Answer: A
A. 25.55%
B. 28.87%
C. 30.12%
D. 31.79%

2. A superheat steam Rankine cycle has turbine inlet conditions of 17.5 MPa and 530 ºC expands in a
turbine to 0.007 MPa. The turbine and pump polytropic efficiencies are 0.9 and 0.7 respectively.
Pressure losses between pump and turbine inlet are 1.5 MPa. What should be the pump work in
kJ/kg? Answer: A
A. 27.13 kJ/kg
B. 29.87 kJ/kg
C. 32.47 kJ/kg
D. 33.25 kJ/kg

3. Steam enters the superheater of a boiler at a pressure of 25 bar and dryness of 0.98 and leaves at the
same pressure at a temperature of 370 ºC. Calculate the heat energy supplied per kg os steam
supplied in the superheater.
Steam properties: at 25 bar and 370 ºC, h = 3171.8 kJ/kg, at 25 bar, h f = 962.11 kJ/kg, hfg = 1841.0
kJ/kg. Answer: A
A. 405.51 kJ/kg
B. 504.15 kJ/kg
C. 154.15 kJ/kg
D. 245.25 kJ/kg

4. A back pressure steam turbine of 100,000 kW serves as a prime mover in a cogeneration system.
The boiler admits the return water at a temperature of 66 ºC and produces the steam at 6.5 MPa and
455 ºC. Steam then enters a back pressure turbine and expands to the pressure of the process, which
is 0.52 MPa. Assuming a boiler efficiency of 80% and neglecting the effect of pumping and the
pressure drops at various location, what is the incremental heat rate for electric? The following
enthalpies have been found: turbine entrance = 3306.8 kJ/kg, exit = 2700.8 kj/kg; boiler entrance =
276.23 kJ/kg, exit = 3306.8 kJ/kg. Answer: A
A. 22,504.23 kJ/kg
B. 52,244.32 kJ/kg
C. 12,435.72 kJ/kg
D. 32,234.82 kJ/kg

5. Steam expands adiabatically in a turbine from 2000 kpa, 400 ºC to 400 kPa, 250 ºC. What is the
effectiveness of the process in percent assuming an atmospheric temperature of 15 ºC. Neglect
changes in kinetic and potential energy.
Steam properties:
At 2000 kPa and 400 ºC 400 kPa and 250 ºC
h = 3247.6 kJ/kg h = 2964.2 kJ/kg
S = 7.1271 kJ/kg -K S = 7.3789 kJ/kg-K. Answer: A
A. 79.62%
B. 84.52%
C. 82.45%
D. 74.57%

6. In an open feedwater heater for a steam power plant, saturated steam at 7 bar is mixed with
subcooled liquid at 7 bar and 25 ºC. Just enough steam is supplied to ensure that the mixed steam
leaving the heater will be saturated liquid at 7 bar when heater efficiency is 90%. Calculate the
mass flow rate of subcooled liquid if steam flow rate is 0.865 kg per second.
Steam properties:
At 7 bar, saturated vapor At 7 bar, saturated liquid
hg = 2763.5 kJ/kg hf = 607.22 kJ/kg
At 7 bar, and 25 ºC
hf = 105.5 kJ/kg. Answer: A
A. 2.725
B. 3.356
C. 2.286
D. 3.948

7. A steam plant operates with an initial pressure of 1.70 MPa and 370 ºC temperature and exhaust to
a heating system at 0.17 MPa. The condensate from the heating system is returned to the boiler at
65.5 ºC and the heating system utilizes from its intended purpose 90% of the energy transferred
from the steam it receives. The turbine efficiency is 70%. If the boiler efficiency is 80%, what is
the cogeneration efficiency of the system in percent. Neglect pumpwork.
Steam Properties:
At 1.7 MPa, and 370 ºC At 1.7 MPa
h = 3787.1 kJ/kg h f = 483.20 kJ/kg
S = 7.1081 kJ/kg-K h fg = 2216.0 kJ/kg
At 65 ºC S f = 1.4752 kJ/kg-K
hf = 274.14 kj/kg S fg = 5.7062 kJ/kg-K. Answer: D
A. 78
B. 102.14
C. 91.24
D. 69

8. In a cogeneration plant, steam enters the turbine at 4 MPa and 400 ºC. One fourth of the steam is
extracted from the turbine at 600 kPa pressure for process heating. The remaining steam continues
to expand to 10 kpa. The extracted steam is then condensed and mixed with feedwater at constant
pressure and the mixture is pumped to the boiler pressure of 4 MPa. The mass flow rate of the
steam through the boiler is 30 kg/s. Disregarding any pressure drops and heat losses in the piping,
and assuming the turbine and pump to be isentropic, how much process heat is required in kW?
Steam Properties:
At 4 MPa, and 400 ºC At 600 kPa
h = 3213.6 kJ/kg h f = 670.56 kJ/kg
S = 6.7690 kJ/kg-K h fg = 2086.3 kJ/kg
Sf = 1.9312 kJ/kg-K
Sfg = 4.8288 kJ/kg-K. Answer: A
A. 15,646.8
B. 2,468.2
C. 3,578.5
D. 1,026.90
9. A 23.5 kg of steam per second at 5 MPa and 400 ºC is produced by a steam generator. The
feedwater enters the economizer at 145 ºC and leaves at 205 ºC. The steam leaves the boiler drum
with a quality of 98%. The unit consumes 2.75 kg of coal per second as received having a heating
value of 25,102 kJ/kg. What would be the over-all efficiency of the unit in percent?
Steam Properties:
At 5 MPa and 400 ºC At 205 ºC
h = 3195.7 kJ/kg hf = 875.04 kJ/kg
At 5 MPa At 145 ºC
hf = 1154.23 kJ/kg hf = 610.63 kJ/kg
hfg = 1640.1 kJ/kg. Answer: C
A. 65
B. 95
C. 88
D. 78

10. A coal fired power plant has a turbine-generator rated at 1000 MW gross. The plant required about
9% of this power for its internal operations. It uses 9800 tons of coal per day. The coal has heating
value of 6,388.9 kCal/kg, and the steam generator efficiency is 86%. What is the net station
efficiency of the plant in percent? Answer: A
A. 33.07%
B. 40.01%
C. 36.74%
D. 30.12%

11. Steam enters the turbine of a cogeneration plant at 7.0 MPa and 500 ºC. Steam at a flow rate of 7.6
kg/s is extracted from the turbine at 600 kPa pressure for process heating. The remaining steam
continues to expand to 10 kPa. The recovered condensates are pumped back to the boiler. The mass
flow rate of steam that enters the turbine is 30 kg/s. Calculate the cogeneration efficiency in
percent.
At 7 MPa, and 500 ºC At 600 kPa
h = 3410.3 kJ/kg h f = 670.56 kJ/kg
S = 6.7975 kJ/kg-K hfg = 2086.3 kJ/kg

At 10 kpa S f = 1.9312 kJ/kg-K

hf = 191.83 kJ/kg Sfg = 4.8228kJ/kg-K.

hfg = 2392.8 kJ/kg

Sf = 0.6493 kJ/kg-K

Sfg = 7.5009 kJ/kg-k. Answer: D

A. 60
B. 50
C. 65
D. 55
12. A 60 MW turbine generator running at 3600 rpm receives steam at 4.0 MPa and 450 ºC with a back
pressure of 10 kPa. Engine efficiency is 78% and the combined mechanical and electrical
efficiency is 95% . What would be the exhaust enthalpy of the steam in kJ/kg? Answer: B
A. 28,124.20 kJ/kg
B. 2,400.12 kJ/kg
C. 20,432.10 kj/kg
D. 30,101.15 kJ/kg

13. Steam enters a throttling calorimeter at a pressure of 1.03 MPa. The calorimeter downstream
pressure and temperature are respectively 0.100 MPa and 125 ºC. what is the percentage moisture
of the supply steam?
Properties of Steam:
P1 = 1.03 MPa hfg = 2010.7 kJ/kg hg = 2779.25 kJ/kg
Note: at 0.100 MPa and 125 ºC, h = 2726.6 kJ/kg. Answer: A
A. 2.62
B. 5.21
C. 3.15
D. 1.98
14. A drum containing steam with 2.5 m in diameter is 7.5 m long. Of the total volume 1/3 contains
saturated steam at 800 kPa and the other 2/3 contains saturated water.if this tank should explode,
how much water would evaporate? Consider the process to be of constant enthalpy. Answer: A
A. 2,948.11 kg
B. 2,424.62 kg
C. 2,651.24 kg
D. 2,123.76 kg

15. A Batangas base industrial company operates a steam-power plant with reheat and regeneration.
The steam enters a turbine at 300 bar and 900 K and expands to 1 bar. Steam leaves the first stage
at 30 bar and part of it entering a closed heater while the rest reheated to 800 K. Both section of the
turbine have adiabatic efficiency of 93%. A condensate pump exist between the main condenser
and the heater. Another pump lies between the heater and condensate outlet line from the heater
(condensed extracted steam). Compute for the extracted fraction of the total mass flow to the
heater. Answer: A
A. 0.234
B. 0.543
C. 0.765
D. 0.485

16. In a Rankine cycle, saturated liquid water at 1 bar is compressed isentropically to 150 bar. First by
heating in a boiler and then by superheating at constant pressure of 150 bar, water substance is
brought to 750 K. after adiabatic reversible expansion in a turbine to 1 bar, it is then cooled in a
condenser to a saturated liquid. How much work is generated in the turbine? Answer: B
A. 967.9 kJ/kg
B. 3976.9 kJ/kg
C. 796.9 kJ/kg
D. 769.9 kJ/kg

17. A reheat steam has 13,850 kPa throttle pressure at the turbine inlet and 2800 kPa reheat pressure,
the throttle and reheat temperature of the steam is 540 °C, condenser pressure is 3.4 kPa, engine
efficiency of high pressure and low pressure is 75%. Find the cycle thermal efficiency. Answer: A
A. 34.46%
B. 35.56%
C. 36.66%
D. 37.76%

18. A simple steam engine plant has boiler feedwater at 95.6 °C. Steam is supplied to the engine at 7.03
kg/cm2, 97% dry. Atmospheric exhaust. Steam rate 9.3 kg per ihp-hr, e m = 87%. Generator of
91.5% efficiency is directly connected to the engine. Determine the Rankine cycle efficiency.
Answer: C
A. 9.88%
B. 12.47%
C. 11.63%
D. 1.055%

19. A 75 W turbine generator has a steam of 12.7 kg/kW-hr. Steam at 12.3 kg/cm 2 gauge 55.6 °C
superheat. Exhaust 50.8 mm Hg abs. find the combined thermal efficiency of a simple vapor cycle
incorporating this unit. Answer: D
A. 9.67%
B. 12.15%
C. 11.64%
D. 10.25%

20. Ab thermal power plant generates 4000 kW and 400 kW for auxiliaries. Determine the net thermal
efficiency if the heat generated by fuel is 15000 kW. Answer: A
A. 24%
B. 26%
C. 28%
D. 30%

21. A steam generating plant has 20 MW turbo-generators. Steam is supplied at 1.7 MPa and 320 °C.
Exhaust is at 0.006 MPa. Daily average load factor is 80%. The steam generating units operate at
70% efficiency when using bunker fuel having a heating value of 31,150 kJ/kg and an average
steam rate of 5 kg steam per kW-hr. Calculate the Mtons of fuel required per 24 hours. Answer: A
A. 514.894 Mtons
B. 5278.496 Mtons
C. 414.894 Mtons
D. 428.496 Mtons

22. Find the entropy of wet steam of temperature 195 °C and dryness 0.90.
Steam Properties:
At 195 °C
Sf = 2.284 kJ/kg-K, Sfg = 4.185 kJ/kg-K. Answer: A
A. 6.05 kJ/kg-K
B. 7.04 kJ/kg-K
C. 5.268 kJ/kg-K
D. 3.575 kJ/kg-K
23. Using the formula S = Cw + + Csup ln , where Cw = mean specific heat of water, Csup =

mean specific heat of superheated steam, T sat = absolute saturation temperature, T = absolute
superheated steam temperature, calculate the entropy per kg of superheated steam at 15 bar (1.5
MPa) and 300 °C, taking the mean specific heats of water and superheated steam as 4.24 kJ/kg-K
and 2.43 kJ/kg-K respectively.
Steam Properties:
At 15 bar (1.5MPa)
Tsat = 198.9 + 273 = 471.3 K
hfg = 1947 kJ/kg . Answer: A
A. 6.9196 kJ/kg-K
B. 5.4972 kJ/kg-K
C. 4.5769 kJ/kg-K
D. 5.4672 kJ/kg-K

24. Dry saturated steam at 5.5 bar (0.55 MPa) is expanded isentropically to 0.2 bar, find the dryness
fraction of the steam at the end of the expansion.
Steam Properties:
At 5.5 bar (0.55 MPa) At 0.20 bar (0.020 MPa)
Sg = 6.790 kJ/kg-K Sf = 0.832 kJ/kg-K
Sfg = 7.075 kJ/kg-K. Answer: D
A. 74.58%
B. 87.58%
C. 84.22%
D. 74.54%

25. Superheated steam at 17 bar (1.7 MPa) and 350 °C is expanded in an engine and the final pressure
is 1.7 bar (0.17 MPa). If the expansion is isentropic, find the dryness fraction of the expanded
steam.
Steam Properties:
At 17 bar (1.7 MPa) and 350 °C At 1.7 bar (0.17 MPa)
S = 7.044 kJ/kg-K Sf = 1.475 kJ/kg-K
Sfg = 5.707 kJ/kg-K. Answer: A
A. 97.59%
B. 89.54%
C. 94.54%
D. 93.54%

26. Superheated steam at a pressure of 20 bar and temperature 300 °C is expanded isentropically. At
what pressure will the steam be just dry and saturated? Answer: A
A. 6 bar
B. 8 bar
C. 10 bar
D. 4 bar

27. Two boilers of equal evaporative capacities generate steam at the same pressure of 15 bar (1.5
MPa) to a common pipe line. One boiler produces superheated steam at 150 °C and the other
produces wet steam. If the mixture is just dry and saturated, find the dryness fraction of the wet
steam from the second boiler.
Steam Properties:
At 15 bar (1.5 MPa), 250 °C At 15 bar (1.5 MPa)
h = 2925 kJ/kg hf = 845 kJ/kg
hfg = 1947 kJ/kg
hg = 2792 kJ/kg. Answer: A
A. 0.9315
B. 0.8425
C. 0.7142
D. 0.6573

28. A 0.2 3 of steam at 7 bar (0.7 MPa) and 0.95 dry is expanded to a pressure of (4.5 bar) 0.45 MPa.
Calculate the dryness fraction of the expanded steam if the expansion follows the law PV = C.
Steam Properties:
At 7 bar (0.7 MPa) At 4.5 bar (0.45 MPa)
3
vg = 0.2728 m /kg vg = 0.4139 m3/kg. Answer: A
A. 0.974
B. 0.846
C. 0.765
D. 0.667

29. A closed system consisting of 1 kg of superheated steam at 20 bar (2 MPa) and 400 °C is cooled at
constant volume until the pressure is 12 bar (1.2 MPa). Find the condition of the steam at the lower
pressure.
Steam Properties:
At 20 bar (2 MPa) At 12 bar (1.2 MPa)
H = 3248 kJ/kg hf = 798 kJ/kg
3
V = 0.1511 m /kg hfg = 1986 kJ/kg
Vg = 0.1632 m3/kg. Answer: B
A. 0.8258
B. 0.9258
C. 0.7258
D. 0.6258

30. The turbine work and the pump work in a Rankine cycle are 1200 kJ/kg and 20 kJ/kg respectively.
What is the efficiency of the cycle if the heat generated by the generstor is 3000 kJ/kg? Answer: B
A. 36.77%
B. 39.33%
C. 35.45%
D. 38.32%

31. In a Rankine cycle, what is the entrance pressure of the turbine if the condenser pressure is 105 kPa
and the pump work is 20 kJ/kg. Use density of water entering the pump, 950 kg/m 3. Answer: C
A. 17.916 MPa
B. 15.601 MPa
C. 19.105 MPa
D. 18.432 MPa

32. Exhaust steam from an engine passes into a condenser at a pressure of 0.12 bar and dryness 0.88.
The temperature of the condensate from the condenser is 40 °C. The circulating water enters the
condenser at 12 °C and leaves at 29 °C. Calculate the mass of circulating water per kg steam
condensed.

Steam Properties:

Steam at 0.12 bar (0.012 MPa)

hf = 207 kJ/kg

hfg = 2383 kJ/kg

Water Properties:

Water at 40 °C Water at 29 °C Water at 12 °C

h = 167.5 kJ/kg h = 121.5 kJ/kg h = 50.4 kJ/kg. Answer:


C

A. 26.71 kg
B. 28.05 kg
C. 30.04 kg
D. 32.86 kg

33. In an experiment to determine the dryness fraction of sream, a simple at a pressure of 1.1 bar (0.11
MPa) was blown into a vessel containing 10 kg of water at 15 °C. The final mass of water mass in
the vessel was 10.75 kg and the final temperature is 55 °C. Find the dryness fraction of steam,
taking the water equivalent of the vessel as 0.45 kg.
Steam Properties:
Steam at 1.1 bar (0.11 MPa)
Hf = 429 kJ/kg
Hfg = 2251 kJ/kg

Water Properties:
Water at 55 °C Water at 15 °C

H = 230.2 kJ/kg h = 62.9 kJ/kg. Answer: A

A. 0.9469
B. 0.8945
C. 0.9156
D. 0.8797

34. The difference in enthalpy at the entrance and exit in the first and second stage of a reheat power
plant are 500 kJ/kg and 900 kJ/kg respectively. Determine the plant cycle efficiency if both stages
has an efficiency of 90% and the heat added to the boiler is 3500 kJ/kg. Neglect pump work.
Answer: A
A. 36%
B. 34%
C. 40%
D. 42%

35. A throttling calorimeter was fitted to pipe carrying steam at 1.2 MPa on order to measure the
dryness fraction. The pressure in the calorimeter was 0.12 MPa and its temperature was 116 °C.
Taking the specific heat of the superheated steam in the calorimeter as 2.0 kJ/kg-K, find the dryness
fraction of the main steam.
Steam Properties:
Steam at 1.2 MPa Steam at 0.12 MPa
Hf = 798 kJ/kg tsat = 104.8 °C
Hfg = 1986 kJ/kg hg = 2683 kJ/kg. Answer: A
A. 0.9603
B. 0.8960
C. 0.6952
D. 0.7563

36. A combined separating and throttling calorimeter was connected to a main steam pipe carrying
steam at 15 bar (1.5 MPa) and following data recorded:
Mass of water collected in a separator = 0.55 kg
Mass of condensate after throttling = 10 kg
Pressure of steam in throttling calorimeter = 1.1 bar = 0.11 MPa
Temperature of steam in throttling calorimeter = 111 °C

Taking the specific heat of the throttled superheated steam as 2.0 kJ/kg-K, find the dryness
fraction of the main steam.

STEAM PROPERTIES:

At 15 bar (1.5 MPa) At 1.1 bar (0.11 MPa)

hf = 845 kJ/kg tsat = 102.3 °C

hfg = 1947 kJ/kg hg = 2680 kJ/kg. Answer: A

A. 0.9018
B. 0.8954
C. 0.7890
D. 0.9872

37. A steam generator has an exit enthalpy of 3500 kJ/kg at the rate of 15 kg/s. Calculate the heat lost
between boiler outlet and turbine inlet if the enthalpy available at the turbine inlet is 3200 kJ/kg.
Answer: A
A. -4500 kW
B. -2500 kW
C. -5700 kW
D. -7650 kW

38. Dry saturated steam at a pressure of (2.4 bar) 0.24 MPa is tapped off the inlet branch of a low
pressure turbine to supply heating steam in a contact feed heater. The temperature of the feedwater
inlet to the heater is 42 °C and the outlet is 99.6 °C. Find the percentage mass of steam tapped off.
Steam Properties:
Steam at (2.4 bar) 0.24 MPa
hg = 2715 kJ/kg
Water Properties
Water at 42 °C Water at 99.6 °C
h = 175.8 kJ/kg h = 417 kJ/kg. Answer: A
A. 9.5%
B. 7.5%
C. 8.5%
D. 7.5%

39. Ten percent of the mass of steam supplied to an engine is bleed off at an intermediate stage and led
to a surface feed heater. The pressure of the steam to the heater is 2.9 bar (0.29 MPa) and the drain
from the heater passes as water to the hotwell. The temperature of the condensate from the
condenser is 40 °C and the temperature of the feedwater to the boilers is 100 °C. Calculate the
dryness fraction of the steam supplied to the heater.
Steam Properties
hf = 556 kJ/kg hfg = 2168 kJ/kg
Water properties
Water at 40 °C Water at 100 °C
h = 167.5 kJ/kg h = 419.1 kJ/kg. Answer: A
A. 0.9809
B. 0.7089
C. 0.8908
D. 0.8498

40. A turbine in a Rankine cycle has an available enthalpy of 1000 kJ/kg. Determine the net cycle
output if the mass flow rate is 6 kg/s and the pump work is 150 kJ/kg. Answer: D
A. 6.8 MW
B. 3.7 MW
C. 4.6 MW
D. 5.1 MW
41. During the process of raising steam in a boiler, when the pressure was 1.9 bar gauge the
temperature inside the boiler was 130 °C and when the pressure was 6.25 bar gauge (0.625 MPag)
the temperature was 165 °C. If the volume of the steam space is constant at 4.25 m 3. Calculate the
mass of steam.
Steam Properties
Steam at 130 °C
P = 2.7 bar
Vg = 0.6686 kg/m3. Answer: D
A. 3.356 kg
B. 4.356 kg
C. 5.356 kg
D. 6.356 kg

42. A heat engine has the following specifications:


Power developed 50 kW
Fuel Burned per hour 3 kg
Heating value of fuel 75,000 kJ/kg
Temperature limits 627 °C and 27 °C. Answer: C
A. Reality
B. Costly
C. Impossible
D. Cheaper

43. A 50 MW thermal plant has a thermal efficiency of 32% based on its combined output. If the plant
operates continuously (24 hr/day). How many tons of coal is burned per day at 30,000 kJ/kg to
produce the desired output. Take boiler efficiency as 85% and how many m 3 per hr sea water (SG =
1.05) is required the condenser if Cpw = 4.2 kJ/kg-K and ∆tw = 10 °C. Answer: A
A. 529.41 tons/day, 8,673.47 m3/day
B. 321.45 tons/day, 5,770.57 m3/day
C. 457.98 tons/day, 7,890.97 m3/day
D. 476.75 tons/day, 4,577.54 m3/day

44. A boiler operates at 1.24 MPa abs and delivers dry and saturated steam, which is expanded to 100
kPa atmospheric pressure in a steam engine having a steam rate of 12 kg per horsepower hr.
Temperature of feedwater is 93 °C. Find the Rankine vapor cycle efficiency. Answer: A
A. 18%
B. 19%
C. 24%
D. 32%

STEAM GENERATOR
1. A steam boiler on a test generates 885,000 lb of steam in a 4 hr period. The average steam pressure
is 400 psia, the average steam temperature is 700 °F, and the average temperature of the feed water
supplied to the boiler is 280 °F. If the boiler efficiency for the period is 82.5% and if the coal has a
heating values of 13,850 Btu/lb as fired, find the average amount of coal burned in short tons per
hour. Answer: B
A. 9.84 short tons per hour
B. 10.75 short tons per hour
C. 12.05 short tons per hour
D. 11.45 short tons per hour

2. A boiler operating at 11 kg/cm2 is required to generate a minimum of 50,000 kg/hr of saturated


steam. Feedwater enters the boiler at 80 °C. The furnace is designed to fire coal at an average rate
of 4,800 kg/hr and boiler efficiency is 85%. Compute the developed boiler horsepower. Answer: A
A. 3,462.52 Hp
B. 4,356.78 Hp
C. 5,234.75 Hp
D. 2,560.45 Hp

3. A water tube boiler evaporated 5.05 kg of water per second from a feed water temperature of
104.44 °C to steam at 1241.1 kPa and quality of 0.97; weight of coal fuel per second 0.57 kg;
higher heating value of coal as fired, 11,800 Btu/lb. Determine the rate of absorption in kJ/s, the
boiler horsepower, and the efficiency of steam generating units. Answer: A
A. 11,558 kJ/kg, 1,178 Bo. Hp, 73.87%
B. 12,345 kJ/kg, 1,234 Bo. Hp, 84.35%
C. 14,567 kJ/kg, 1,045 Bo. Hp, 67.45%
D. 10,275 kJ/kg, 1,475Bo. Hp, 83.25%

4. A waste heat recovery boiler produces 4.8 (dry saturated) steam from 104 °C feed water. The boiler
receives energy from 5 kg/s of 954 °C dry air. After passing through the waste heat boiler, the
temperature of the air has been reduced to 343 °C. How much steam in kg is produced per second?
Note: At 4.8 MPa dry and saturated, h = 2796.0 kJ/kg. Answer: A
A. 1.3
B. 0.92
C. 2.1
D. 3.4

5. A steam generating plant consisting of a boiler, an economizer and super heater generates
superheated steam at the rate of 50 tons per hr. feedwater enters the boiler at 5 MPa and 120 °C.
Steam leaves the superheater at 4.5 MPa and 320 °C. if the coal used has a heating value of 30,000
kJ/kg, calculate the no. of tons of coal fired per hr for a gross efficiency of 85%. Answer: A
A. 4.889 tons/hr
B. 5.897 tons/hr
C. 3.267 tons/hr
D. 2.675 tons/hr

6. A water tube boiler has a capacity of 1000 kg/hr of steam. The factor of evaporation is 1.3, boiler
rating is 200%, boiler efficiency is 65%, heating surface area is 0.91 m 2 per Bo. Hp, and the heating
value of fuel is 18,400 kCal/kg. The total coal available in the bunker is 50,000 kg. Determine the
no. of hours to consume the available fuel. Answer: A
A. 853.36 hrs
B. 706.57 hrs
C. 979.46 hrs
D. 100.75 hrs
7. Two boilers are operating steadily on 91,000 kg of coal contained in a bunker. One is producing
1591 kg of steam per hour at 1.2 factor of evaporation and an efficiency of 65% and another boiler
produced 1364 kg of steam per hour at 1.15 factor of evaporation and an efficiency of 60%. How
many hours will the coal in the bunker run the boilers if the heating value of coal is 7,590 kCal/kg?
Answer: A
A. 230.80 hrs
B. 280.54 hrs
C. 350.35 hrs
D. 300.54 hrs

8. The following coal has the following ultimate analysis by weight:


C = 70.5% H2 = 4.5% O2 = 6.0% Moisture = 4%
S = 3.0% Ash = 11% N2 = 1.0%
A stocker fired boiler of 175,000 kg/hr steaming capacity uses this coal as fuel. Calculate the
volume of air in m3/hr with air at 60 °F and 14.7 psia pressure if boiler efficiency is 70%, e = 30%
and FE = 1.10. Answer: A
A. 212,830 m3/hr
B. 342,987 m3/hr
C. 122,345 m3/hr
D. 132,456 m3/hr

9. Find the rated boiler horsepower of a H.R.T. boiler 60 in. in diameter 16 ft long, and having
seventy and 3 in. O.D. tubes with 0.109 in. wall? Answer: A
A. 95.09 Bo. Hp
B. 78.09 Bo.Hp
C. 89.09 Bo. Hp
D. 93.09 Bo. Hp

10. What is the heating surface of a water tube boiler if the equivalent rated boiler horsepower is 200?
Answer: A
A. 182
B. 206
C. 450
D. 198

11. The heating surface area of a fire tube boiler is 400 m 2. What is the equivalent rated boiler
horsepower? Answer: A
A. 363.64 Hp
B. 435.72 Hp
C. 256.78 Hp
D. 135.72 Hp
12. What would be the percentage of the over-all efficiency of a steam generator obtaining an
evaporation equivalent to 10.95 kg of water from and at 100 ºC per kg of coal containing 34,064
kJ/kg heating value? Answer: A
A. 73%
B. 74%
C. 82%
D. 70%

13. For a generation of dry and saturated steam at 1.0 MPa absolute, what is the percentage gain in heat
when the boiler feedwater is heated from 30 ºC to 90 ºC? Answer: C
A. 8.47%
B. 9.97%
C. 9.47%
D. 5.47%

14. What is the percent rating of a water tube boiler if the heating surface area is 400 m 2 and the
developed boiler horsepower is 750? Answer: A
A. 170.625%
B. 140.675%
C. 130.625%
D. 120.765%

15. Coal with a higher heating value of 6700 kCal/kg is consumed at the rate of 600 kg/hr in a steam
generator with a rated boiler horsepower of 200. The feedwater temperature is 82 ºC and steam
generated is at 10.2 kg/cm 2 abs. saturated condition. The horsepower developed is equivalent to
305. What is the heating surface area? Answer: A
A. 182 m2
B. 230 m2
C. 123 m2
D. 324 m2

16. Based on problem no. 15, calculate the rate of steam evaporated. Answer: A
A. 4,424.68 kg/hr
B. 4,567 kg/hr
C. 5,579.78 kg/hr
D. 5.678.45 kg/hr

17. Based on problem no. 15, What is the percent rating developed? Answer: A
A. 152.50%
B. 125.45%
C. 234.75%
D. 134.57%

18. Based on problem no. 15, determine the ASME evaporation unit in kBtu/hr. Answer: A
A. 10,511.76 kBtu/hr
B. 9,234.75 kBtu/hr
C. 10,211.57 kBtu/hr
D. 12,377.75 kBtu/hr

19. Based on problem no. 15, what is the factor of evaporation? Answer: A
A. 1.08
B. 1.52
C. 1.14
D. 1.45

20. Based on problem no. 15, calculate the over-all thermal efficiency|? Answer: A
A. 64.02%
B. 60.24%
C. 67.24%
D. 72.13%

21. Based on problem no. 15, determine the actual specific evaporation in kg per kg coal. Answer: A
A. 7.37 kgsteam/kgcoal
B. 7.57 kgsteam/kgcoal
C. 7.23 kgsteam/kgcoal
D. 6.24 kgsteam/kgcoal

22. Based on problem no. 15, determine the equivalent specific evaporation in kg per kg coal. Answer:
A
A. 7.95 kgsteam/kgfuel
B. 8.37 kgsteam/kgfuel
C. 7.45 kgsteam/kgfuel
D. 7.85 kgsteam/kgfuel

23. An auxiliary boiler produces steam at 8 bar (0.80 MPa), 0.98 dry from feedwater at 43 ºC, and fuel
of calorific value of 42 MJ/kg is burned at the rate of 1.2 tons per day. Assuming an efficiency of
70%, calculate the steam production per hour. Answer: A
Steam at 8 bar (0.80 MPa)
hf = 721 kJ/kg hs = hf + hfg
hfg = 2048 kJ/kg 721+0.98(2048)
2728 kJ/kg
Water at 43 ºC: h = 43(4.187) = 180.04 kJ/kg
A. 576.93 kg/hr
B. 476.86 kg/hr
C. 376.45 kg/hr
D. 275.35 kg/hr

24. During a test on an oil-fierd boiler the folling data were recorded:
Pressure of steam 2.5 MPa
Temperature of steam 300 ºC
Mass of feedwater entering the Boiler 11350 kg/hr
Temperature of Feedwater 100 ºC
Mass of fuel burned 875 kg/hr
Calorific value 42.3 MJ/kg
Steam at (25 bar) 2.5 MPa, 300 ºC h = 3010 kJ/kg
Water at 100 ºC h = 419.10 kJ/kg
Calculate the efficiency of the boiler. Answer: A
A. 79.49%
B. 87.49%
C. 53.49%
D. 34.57%
25. Fuel of calorific value 43.5 MJ/kg is burned in an auxiliary boiler at the rate of 81 kg/hr. The
feedwater rate is 0.258 kg/s and its temperature is 48 ºC. If the efficiency of the boiler is 68% and
the steam produced is dry and saturated, estimate its pressure. Answer: A
A. 1.1 MPa
B. 1.5 MPa
C. 2.0 MPa
D. 2.1 MPa

26. The equivalent evaporation of a boiler, from and at 100 ºC is 15 kg steam per kg fuel and the
calorific value of the fuel burned is 41.9 Mj/kg. Find the efficiency of the boiler. Answer: A
A. 80.81%
B. 78.96%
C. 82.3%
D. 77.87%

27. What is the developed boiler horsepower if the steam rate is 0.85 kg/s and the factor of evaporation
is 1.25. Answer: A
A. 244.41Hp
B. 214.54 Hp
C. 200.57 Hp
D. 157.25 Hp

28. The heating value of fuel suppled in a boiler is 40,000 kJ/kg. If the factor of evaporation is 1.10
and the actual specific evaporation is 10, what is the efficiency of the boiler? Answer: A
A. 62.07%
B. 53.08%
C. 787.05%
D. 54.97%

29. Steam at 30 bar (3 MPa), 375 ºC is generated in a boiler at the rate of 30,000 kg/hr from feed water
at 130 ºC. the fuel has a calorific value of 42 MJ/kg and the daily consumption is 53 tons. Calculate
the boiler efficiency.
Steam Properties:
Steam at 30 bar (3 MPa), 375 ºC: h = 314 kJ/kg
Water at 130 ºC: h = 546 kJ/kg. Answer: A
A. 85%
B. 75%
C. 88%
D. 95%

30. A boiler contains 3.5 tons of water initially having 40 ppm dissolved solids and after 24 hrs the
dissolved solids in the water is 2500 ppm. If the feed rate is 875 kg/hr, find the ppm of dissolved
solids contained in the feed water. Answer: A
A. 410 ppm
B. 320 ppm
C. 390 ppm
D. 420 ppm
31. A fuel of Qh = 6,888 kCal was burned in the furnace of 223 m 2 water tube boiler which was
generating 5,443 kg steam per hour at 14.06 kg per cm 2 abs. dry and saturated, from 109 ºC feed, at
a time when fuel consumption was 612 kg per hour. Find the percent rating. Answer: A
A. 146%
B. 150%
C. 230%
D. 157%

32. Estimate the required furnace volume for a pulverized coal-fired steam generator where 1375 x 10 4
kCal per hour of heat transfer at 80% thermal efficiency is the expected thermal performance.
Boiler has water walls. Ash fusion temperature is 1204 ºC and the design value of furnace heat
release rate is 195,800 kCal per m3 per hour. Answer: A
A. 87.79 m3
B. 89.56 m3
C. 82.35 m3
D. 81.45 m3

33. Two boilers has capacities of 200 kg/hr and 250 kg/hr. if 150,000 kg of coal are supplied to two
boilers, how many days to consume the available fuel? Answer: A
A. 13.89 days
B. 17.87 days
C. 15.78 days
D. 24 days

34. What is the net boiler efficiency if the heat generated by fuel is 40,000,000 kJ/hr and the boiler
auxiliaries consumes 2 MW. The ASME evaporation units of a boiler is 25,900,000 kJ/hr. Answer:
A
A. 46.75%
B. 36.75%
C. 56.75%
D. 86.75%

35. Testing of a stoker-fired boiler reveal that, during a period when 4550 lb of 12,400 BTU coal were
burned, refuse accumulated in the ashpit to the extent of 495 lb, although the coal as fierd
contained only 10% ash by analysis. Determine the grate efficiency if this stoker considering no
fly-ash. Answer: A
A. 98.97%
B. 92.35%
C. 95.87%
D. 90.21%

36. A travelling-grate stoker can burn bituminous coal with 10% moisture and 10% ash at a rate of
500,000 BTU/hr-ft2. A boiler with a steam rating of 200,000 lb/hr will be fired with the above fuel
having a high heat value of 12,200 BTU/lb. if the boiler efficiency is 80%, and if it takes 1000
BTU to evaporate and superheat one pound of feedwater to the desired temperature, find the hourly
coal supply and grate area neened. Answer: A
A. 20,500 lb, 500 ft-2
B. 26,400 lb, 700 ft2
C. 28,200 lb,400 ft2
D. 22,600 lb, 200 ft2
37. What is the rate of evaporation of a water tube boiler if the factor of evaporation is 1.10, percent
rating of 200% and the heating surface area is 250 m 2? Answer: A
A. 7,817.16 kg/hr
B. 7,898.67 kg/hr
C. 6,789.45 kg/hr
D. 5,768.54 kg/hr

38. An oil-fired steam boiler generates 13,000 lb of steam per hour at 300 psia, 650 ºF when the fuel is
burned at 120 gallons per hour. Feedwater, 300 ºF. calculate the thermal efficiency and equivalent
evaporation per lb of fuel oil. Answer: A
A. 77.09% & 15.4 lbsteam/lbfuel
B. 87.09% & 19.7 lbsteam/lbfuel
C. 79.07% & 12.5 lbsteam/lbfuel
D. 89.07% & 18.8 lbsteam/lbfuel
39. One hundred thousand pounds of water per hour at 750 psia and 250 ºF (h f = 230.9 BTU/lb) are
received by a steam generating unit. Steam leaves the superheater at 650 psia and 600 ºF (h s =
1286.5 BTU/lb). the fuel used is coal whose heating value is 12,500 BTU/lb. Calculate the weight
of fuel fired per hour if economizer efficiency is 85%. Answer: A
A. 4.44 long tons per hour
B. 5.44 long tons per hour
C. 3.43 long tons per hour
D. 1.45 long tons per hour

40. A fire-tube boiler operates at 200 psia (1.378 MPa) with 200 ºF (93.3 ºC) feedwater, both at
saturated condition. The diameter of the boiler is 213.4 cm, length of 5.5 m, and having 84 tubes,
10.2 cm inside diameter, 10.5 cm outside diameter. Determine the boiler heating surface in m 2 and
the weight of steam generated in kg/s if the boiler evaporates 34.5 lb/hr of water per 12 ft 2 of
heating surface or 3.9 g/m2-s? Answer: A
A. 2,360.67 kg/hr
B. 1,234.57 kg/hr
C. 3,247.76 kg/hr
D. 4,576.68 kg/hr

41. Wet steam is being generated by a boiler at the rate of 1500 kg per hour from 26.7 ºC feed water.
Quality of the steam is 98% and gauge shows a pressure of 9 kg/cm 2. Calculate the equivalent
evaporation and the heat absorbed by the boiler per hour in kCal when producing steam? Answer:
A
A. 1743.72 kg/hr
B. 2376.42 kg/hr
C. 1897.74 kg/hr
D. 1235.76 kg/hr

42. What is the total pressure loss in steam pipe 164 m long if the equivalent pressure loss is 50 kPa
per 30 meters? Answer: A
A. 273 kPa
B. 173 kPa
C. 372 kPa
D. 473 kPa

43. A high temperature source at 950 K provides 580 kJ heat to a heat engine. The heat engine converts
200 kJ net work and rejects the balance to a temperature sink at 295 K. compare the thermal
efficiency Et of this engine to the thermal efficiency Erev of the Carnot cycle reversible heat engine.
Answer: C
A. Et = Erev
B. Et > Erev
C. Et < Erev
D. Et ≤ Erev
44. A 150 grams of water at 75 ºC is heated at constant pressure. The water is completely vaporized.
Find the heat added. Answer: C
A. 543.52 kJ
B. 816.66 kJ
C. 354.25 kJ
D. 1135.66 kJ

STEAM PRIME MOVERS


1. Steam is admitted to the cylinder of an engine in such a manner the average pressure is 120 psi. the
diameter of the piston is 10 in. and the length of stroke is 12 in. What is the Hp of the engine when
it is making 300 rpm? Answer: A
A. 171.5 Hp
B. 175 Hp
C. 173.2 Hp
D. 174.4 Hp

2. The area of an indicator card taken off the high pressure cylinder of a quadruple expansion engine
is 1445 mm2, the length is 80 mm, and 1 mm of height represents 0.27 bar. The bore of the cylinder
runs at 1.65 rev/sec. Calculate the total indicated power of the engine assuming equal powers are
developed in all cylinders. Note: length of stroke 105 mm and diameter of 600 mm. Answer: A
A. 1911.2 kW
B. 2345.7 kW
C. 1123.5 kW
D. 2331.43 kW

3. During a test on a single-cylinder steam engine, the mean effective pressure was 405 kPa when
running at 3 rev/sec. The brake load was 1.9 kN acting at an effective radius of 1.2 m. If the
cylinder diameter is 0.275 m and stroke 0.4 m, calculate the mechanical efficiency. Answer: A
A. 74.46%
B. 84.54%
C. 73.24%
D. 23.45%

4. A steam engine has anindicated efficiency of 65%. The enthalpies of the engine at entrance and exit
are 3000 kJ/kg and 2500 kJ/kg respectively. If the mechanical efficiency is 85% and the steam
consumption is 1000 kg/hr, what is thebrake power of the engine? Answer: A
A. 76.64kW
B. 78.54 kW
C. 87.67 kW
D. 82.34 kW
5. Steam at 7.5 bar is admitted to an engine cylinder and cut-off at 0.34 of the stroke. The back
pressure is 1.7 bar. Neglecting clearance, calculate the hypothetical mean gross pressure and mean
effective pressure. Answer: A
A. 5.301 bar & 3.601 bar
B. 5.601 bar & 2.602 bar
C. 5.467 bar & 2.457 bar
D. 4.578 bar & 3.4547 bar

6. Steam is supplied to the cylinder of an engine at 8.3 bar and cut-off takes place when the piston has
traveled 0.45 of its stroke. The clearance volume is equal to 5% of the stroke volume. If the back
pressure is 2.4 bar, calculate the actual mean effective pressure taking a diagram factor of 0.72.
Answer: A
A. 3.178 bar
B. 3.478 bar
C. 2.457 bar
D. 2.574 bar

7. The mean effective pressure of an engine running at 300 rpm is 500 kPa. What is the indicated
power if the engine has bore of 250 mm x 450 mm. Answer: A
A. 110.5 kW
B. 210.7 kW
C. 123.5 kW
D. 312.4 kW

8. The cylinder of a triple expansion engine are 600, 950 and 1500 mm diameter and the stroke is
1050 mm. When running at 1.8 rev/s, the mean effective pressures in the cylinders are 517, 207 and
83 kPa respectively. Calculate the total indicated power of the engine. Answer: A
A. 1661.8 kW
B. 2345.7 kW
C. 2457.7 kW
D. 1758.34 kW

9. Steam at 1.5 MPa, 300 ºC is supplied to an engine and exhaust at 0.014 MPa. The consumption of
steam is 28,500 kg/hr when the indicated power is 3000 kW. Calculate the steam consumption on
kg/kW-hr and the indicated thermal efficiency. Answer: A
A. 9.5 kg/kW-hr, 13.45%
B. 8.5 kg/kW-hr, 12.34%
C. 10 kg/kW-hr, 11.45%
D. 10.5 kg/kW-hr, 10.47%

10. A steam engine delivers 1000 kg/hr steam. The enthalpies at entrance and exit of the engine
are3000 kJ/kg and 400 kJ/kg respectively. Calculate the brake power of the engine if the brake
thermal efficiency is 20%. Answer: A
A. 144.44 kW
B. 200.45 kW
C. 254.34 kW
D. 345.75 kW

11. An engine is supplied with steam at a pressure 1.5 MPa and temperature 250 ºC, and pressure of
the exhaust is 0.016 MPa. Assuming isentropic expansion find the dryness fraction of the steam
after expansion. Answer: A
Steam Properties:
At 1.5 MPa, 250 ºC
S = 6.711 kJ/kg=K
At 0.016 MPa:
Sf = 0.722 kJ/kg-K
Sfg = 7.213 kJ/kg-K
A. 0.8235
B. 0.8978
C. 0.7689
D. 0.7654

12. A double acting steam engine with bore and stroke of 250 mm x 450 mm rotates 250 rpm. If the
mean effective pressure is 400 kPa, what is the indicated power developed in the cylinder?
Answer: A
A. 73.63 kW
B. 78.57 kW
C. 84.75 kW
D. 85.57 kW

13. In a test laboratory, it was found out that of the 80 BHP developed by the engine on test, 45 Hp are
absorbed by the cooling water that is pumped through the water jacket and the radiator. The water
enters the top of the radiator at 200 ºF. At that temperature, enthalpy of water is 168.07 BTU/lb.
Water leaves the bottom of the radiator at 190 ºF and with an enthalpy of 158.03 BTU/lb. What is
the water flow rate of steady-state operation? Answer: A
A. 22.8 gal/min
B. 52.3 gal/min
C. 12.58 gal/min
D. 18.95 gal/min

STEAM TURBINE (sample problems)


14. An industrial plant requires 1.5 kg of dry saturated steam per second at 165 ºC for heating
purposes. This steam may be supplied from an extraction turbine which receives steam at 4 MPa
and 380 ºC and is exhaust to condenser at the rate of 0.80 kg steam per second at 0.0034 MPa while
rejecting 1400 kW to the cooling water. If the mechanical-electrical efficiency is 95% and that the
heat loss in the turbine casing is 10 kW, calculate the power generated by the plant. Answer: A
A. 1556.5 kW
B. 1879.7 kW
C. 1879.45 kW
D. 1489.55 kW

15. Steam enters a turbine stage with an enthalpy of 3628 kJ/kg and a velocity at 70 m/s and leaves the
same stage with an enthalpy of 2846 kJ/kg and a velocity of 124 m/s. Calculate the power if there
are 5 kg/s steam admitted at the turbine throttle? Answer: C
A. 4597.45 kW
B. 3976.55 kW
C. 3883.81 kW
D. 1675.42 kW

16. An adiabatic turbine in a steam generating plant receives steam at a pressure of 7 MPa and 550 ºC
and exhaust at 20 kPa. The turbine inlet is 3 m higher than the turbine exit, the inlet steam velocity
is 15 m/s and the exit velocity is 300 m/s. Calculate the turbine work per unit mass of steam.
Answer: A
A. 1196.60 kJ/kg
B. 1249.78 kJ/kg
C. 1578.5 kJ/kg
D. 1675.42 kJ/kg

17. A steam turbine with an internal efficiency of 90 percent receives steam at 7 MPa 550 ºC and
exhaust at 20 kPa. Determine the ideal turbine work, exhaust enthalpy and the exit quality of steam.
(Use the properties of steam in problem no. 16). Answer: A
A. 1241.22 kJ/kg, 2413.802 kJ/kg, 91.69%
B. 1151.22 kJ/kg, 2413.802 kJ/kg, 91.69%
C. 1241.22 kJ/kg, 2513.802 kJ/kg, 92.69%
D. 1421.22 kJ/kg, 2143.802 kJ/kg, 96.91%

18. Steam with an enthalpy of 800 kCal/kg enters a nozzle at a velocity of 80 m/s. Find the velocity of
the steam at the exit of the nozzle if its enthalpy is reduced to 750 kCal/kg, assuming the nozzle is
horizontal and disregarding heat losses.
Take g = 9.81 m/s2 and J constant = 427 kg-m/kCal. Answer: C
A. 452.37 m/s
B. 245.45 m/s
C. 651.92 m/s
D. 427.54 m/s

19. Steam is expanded through a nozzle and the enthalpy drop per kg of steam from the initial pressure
to the final pressure is 60 kJ. Neglecting friction, find the velocity of discharge and the exit area of
the nozzle to pass 0.20 kg/s if the specific volume of the steam at exit is 1.5 m 3/kg. Answer: D
A. 346.4 m/s, 879 m2
B. 356.7 m/s, 278 m2
C. 765.6 m/s, 467 m2
D. 346.4 m/s, 866 m2

20. The entrance and exit enthalpies of a steam turbine are 4000 and 3000 kJ/kg respectively. What is
the mass of steam entering the turbine if the generator power is 3 MW and generator efficiency is
90%. Answer: A
A. 12,000 kg/hr
B. 14,000 kg/hr
C. 17,000 kg/hr
D. 24,000 kg/hr

21. Dry saturated steam at 8 bar (0.8 MPa) is expanded in turbine nozzle to a pressure of 5 bar 0.97
dry. Find the velocity at exit. If the area at exit is 14.5 cm 2, find the mass flow rate of the steam in
kg/s. Answer: A
Steam Properties:
At 5 bar; (0.5 MPa) At 8 bar (0.8 MPa)
hf = 640 kJ/kg v = 0.3748 m3/kg
hg = 2769 kJ/kg
hfg = 2109 kJ/kg
A. 1.625 kg/s, 407.4 m/s
B. 2.456 kg/s, 786.5 m/s
C. 2.257 kg/s, 233.8 m/s
D. 3.125 kg/s, 125.7 m/s

22. A steam turbine has an entrance enthalpy of 3000 kJ/kg and an exit enthalpy of 2000 kJ/kg. What is
the stage efficiency if the actual enthalpy after isentropic expansion is 2,200 kJ/kg? Answer: D
A. 82%
B. 70%
C. 86%
D. 80%

23. In a Rankine cycle, steam turbine with exhaust enthalpy of 2500 kJ/kg delivers 0.50 kg/s of steam.
If the exit enthalpy of the condenser is 500 kJ/kg, what is the heat rejected? Answer: A
A. 1000 kW
B. 1200 kW
C. 1700 kW
D. 1500 Kw

24. Steam enters the nozzle of an impulse turbine at 2 MPa, 300 ºC and leaves at 0.30 MPa, 0.98 dry.
The angle of the nozzles is 20º to the direction of motion of the blades and blade velocity is 380
m/s. Calculate the velocity of the steam leaving the nozzles. Answer: A
A. 829.4 m/s
B. 924.8 m/s
C. 545.7 m/s
D. 754.8 m/s

25. Steam enters the turbine at the rate of 11,000 kg/hr with enthalpies of 3000 kJ/kg and 1000 kJ/kg at
the entrance and exhaust respectively. Steam is extracted from the turbine at 3500 kg/hr for heating
purposes with enthalpy of 2700 kJ/kg. Calculate the turbine work. Answer: C
A. 5,468.33 kW
B. 4,569.33 kW
C. 4,458.33 kW
D. 5,425.33 kW

26. After checking on the properties and phase descriptions of water, the quality is found to be x =
0.70. How many percent of the mass is in the liquid phase? Answer: D
A. 70%
B. 0.70%
C. 100%
D. 30%

27. An engine requires 4300 k of dry steam per hour. The quality of steam is 90%. What must be the
weight of steam? Answer: A
A. 4778 kg/hr
B. 4000 kg/hr
C. 5258 kg/hr
D. 6000 kg/hr

28. The pressure and temperature entering the turbine are 1800 kPa and 380 ºC. The pressure leaving
the turbine is 20 kPa. The quality of the steam entering the condenser is 90%. What is the turbine
work? Answer: D
Steam Properties: @ 1800 kPa and 380 ºC (h = 3207.2 kJ/kg)
@ 20 kPa, hf = 251.4 kJ/kg and hg = 2609.7 kJ/kg
A. 1833.33 kJ/kg
B. 1333.8 kJ/kg
C. 597.5 kJ/kg
D. 833.33 kJ/kg

29. A 6 MW steam turbine generator power plant has a full-load steam rate of 8 kg/kW-hr. assuming
that no-load steam consumption as 15% of full-load steam consumption, compute for the hourly
steam consumption at 75% load, in kg/hr. Answer: A
A. 37,800 kg/hr
B. 38,700 kg/hr
C. 30,780 kg/hr
D. 30,870 kg/hr

30. A 4 kg air enters a turbine with enthalpy of 600 kJ and velocity of 250 m/s. The enthalpy at exit is
486 kJ and velocity of 170 m/s. What is the work developed if there is a heat loss of 10 kJ?
Answer: B
A. 128.83 kJ
B. 171.2 kJ
C. 80.2 kJ
D. 28.3 kJ

GAS LOOP
1. Given a horizontal conveyor, 46 m centers, 175 pounds per hour capacity of handling bituminous
coal at 0.5 m/s with 800 kg/m3. Other data as follows:
Flight width and depth 610 mm x 200 mm
Quality of material 0.108 m3/m
Coefficient of friction elements 0.10
Material Coefficient of friction 0.59
Assume an engineering-type chain with sleeve bearing rollers weighing with flights, 89.3 kg/m.
Calculate the chain pull in kg. Answer: D
A. 2180.33
B. 4550.10
C. 1555.36
D. 3166.46

2. What is the power in kW required to drive a horizontal conveyor 46 m centers, 175 tons/ hr
capacity of bituminous coal at 0.5 m/s with 800 kg/m3 material?
Other data are as follows:
Flight width and depth 610 mm x 200 mm
Quality of material 0.108 m3/m
Coefficient of friction elements 0.10
Material Coefficient of friction 0.59
Assume an engineering-type chain with sleeve bearing rollers weighing with flights, 89.3 kg/m.
Let Fp = 76.042 kg-m/Hp-s. Answer: C
A. 10.20 kW
B. 20.50 kW
C. 15.53 kW
D. 30.15 kW

3. A chain grate stoker 9.5 ft, wide, receives crushed coal having a mean specific weight of 54 lb/ft 3.
The coal is spread to a uniform depth of 6.8 in. by the coal gate. Determine the stoker speed in fpm
if the furnace is to burn 7.5 tons per hour. Answer: A
A. 0.86 ft/min
B. 098 ft/min
C. 1.86 ft/min
D. 1.98 ft/min

4. Find the length of a suspension bunker to contain 181 tons of coal without surcharge; width, 4.6 m;
depth 4.3 m. the level capacity of a suspension bunker is 5/8 wdL: where w = width, d = depth and
L = length. Density of coal, 800 kg/m3. Answer: A
A. 18.30 m
B. 123.80 m
C. 17.61 m
D. 12.61 m

5. Calculate the belt width of a conveyor to transport 18 tons per hour at 20 º length sufficient to give
33.5 m rise. 1750-rpm motor, V-velt drive to helical gear speed reducer. Belt speed, 61 m/min.
discharge over tail pulley. Answer: B
A. 25.41 cm
B. 27.03 cm
C. 35.41 cm
D. 37.03 cm

6. The individual buckets of a vertical elevator carry 3.6 kg coal and are spaced 30.5 cm apart on the
chain. Sprocket wheels 61 cm diameter, chain speed 79.2 m/min. heght between sprockets, 9.1 m.
drive: 1750-rpm motor through sprockets and steel roller chain. What is the motor horsepower
required to drive the conveyor drive shaft? Answer: C
A. 2 Hp
B. 4 Hp
C. 5 Hp
D. 3 Hp

7. An open coal yard has dimensions 36.6 m x 15.2 m. how many tons of coal can be stockpiled on it
to a depth of 3.05 m sides slopes 35º. Answer: C
A. 804.26 tons
B. 1000.62 tons
C. 900.12 tons
D. 754.79 tons

8. Find the draft loss in rectangular breeching 2.59 m x 30.48 m long when 196 m 3/s flue gas of 0.769
kg/m3 density are flowing. Answer: D
A. 2.580 cm H2O
B. 2.086 cm H2O
C. 1.580 cm H2O
D. 1.085 cm H2O

9. A forced draft fan when driven by a constant speed motor produces a pressure of 20.32 cm water
when Q = 1415.8 m3/min. what pressure would it produce on same air if Q were made 1165.4
m3/min. What would be the relative speed and power? Answer: A
A. 52 cm, of H2O, 1.60 N1, 4.1 P1
B. 50 cm, of H2O, 1.80 N1, 4.0 P1
C. 45 cm, of H2O, 1.60 N1, 4.1 P1
D. 47 cm, of H2O, 1.81 N1, 4.1 P1

10. The products of combustion of 635 kg coal per hour pass through a breeching at 260 ºC and 7.62
m/s. Excess air, 50%. Assume square section breeching and find its dimensions. Sea level, and
density of the flue gas-coal fired = 0.68 kg/m3. Answer: A
Ultimate analysis of coal is as follows:
Sulfur 0.55% Nitrogen 1.17%
Hydrogen 4.5% Oxygen 6.03%
Carbon 84.02% Ash 3.73%
A. 77 cm x 77 cm
B. 66 cm x 66 cm
C. 88 cm x 88 cm
D. 55 cm x 55 cm

FEED WATER LOOP


1. The feedwater to a boiler is 92% condensate and 8% make-up containing 270 ppm solids. What
weight of solids enter the boiler per hour at 22,680 kg/hr steam evaporation. Answer: A
A. 0.49 kg solids/hr
B. 059 kg solids/hr
C. 0.39 kg solids/hr
D. 0.69 solids/hr
2. He 1370 diameter steam drum on a boiler is 2440 mm long has a 250 mm gauge glass at mid-drum
level. Find the maximum steam generation that could be cared for by a blowdown of half a water
gauge each 8 hr. shift. Pressure, 17.5 kg/cm2 gage. Sf = 150 ppm, Sb = 2000 ppm. Answer: B
A. 450.49 kg/hr
B. 550.59 kg/hr
C. 650.59 kg/hr
D. 750.59 kg/hr

3. The exhaust of a mechanical drive turbine is estimated to be 1270 kg/hr at 14 kg/cm 2 gage, and
2850 J/g. How much feedwater will this heat per hour with 2.7 ºC terminal difference (tdº)? Cold
water temperature, 26.7 ºC Answer: C
A. 18,146.04 kg/hr
B. 16,148.04 kg/hr
C. 11,468.04 kg/hr
D. 14,168.04 kg/hr

4. Estimate discharge rate (L/min) and maximum suction lift of a 95 mm x 127 mm x 152 mm duplex,
direct-acting steam pump. Ev = 0.90 sea level. Answer: D
A. 187.90 L/min
B. 287.90 L/min
C. 487.90 L/min
D. 387.90 L/min

5. Calculate drive horsepower for pumping 1703 L/min cold water to a tank suction at 127 mm Hg
vacuum, delivery at 5.3 kg/cm2 ga., both measured close to pump, ep = 065. Answer: A
A. 31.42 Hp
B. 20.42 Hp
C. 35.42 Hp
D. 23.02 Hp

GEOTHERMAL POWER PLANT


1. A liquid dominated geothermal plant with a single flash separator receives water at 204 ºC. The
separator pressure is 1.04 MPa. A direct contact condenser operates at 0.034 MPa. The turbine has
a polytropic efficiency of 0.75. For a cycle output of 50 MW, what is the mass flow rate of the
well-water in kg/s? Answer: A
A. 2100
B. 2871
C. 1867
D. 2444

2. A geothermal power plant draws a pressurized water from well at 20 MPa and 300 ºC. to produce a
steam-water mixture in the separator, where the unflashed water is removed, this water is throttled
to a pressure of 1.5 MPa. The flashed steam which is dry and saturated passes through the steam
collector and enters the turbine at 1.5 MPa and expands to 1 atm. The turbine efficiency is 85% at a
rated power output of 10 MW. Calculate the over-all plant efficiency. Answer: A
A. 7.26%
B. 5.28%
C. 10.7%
D. 12.7%
3. A flashed steam geothermal power plant is located where underground hot water is available as
saturated liquid at 700 kPa. The well head pressure is 600 kPa. The flashed steam enters a turbine
at 500 kPa and expands to 15 kPa, when it is condensed. The flow rate from the well is 29.6 kg/s.
Determine the power produced in kW. Answer: D
A. 453.16 kW
B. 540.36 kW
C. 254.25 kW
D. 430.16 kW

4. In a certain geothermal area, studies show that 1,500,000 kg/hr of pressurized ground water is
available at 2500 psia and 620 ºF the water will be throttled to 250 psia to produce wet steam and
this mixture will be passed through a water separator to remove the water droplets so that saturated
steam at 250 psia is available at the entrance of steam turbine for the proposed power plant. Other
data are as follows:
Discharge pressure of turbine 25 in. Hg vacuum
Turbine Engine efficiency 75%
Mechanical Loss 1.5% of shaft power
Generator efficiency 97%
Assume atmospheric pressure to be 30 in Hg. Determine the maximum amount of power in kW
that the plant can generate. Answer: C
A. 53.16 MW
B. 80.36 MW
C. 68.06 MW
D. 43.16 MW

5. A flashed steam geothermal plant has pressurized underground water available at 102 kg/cm 2 and
240 ºC. in order to produce steam-water mixture, the ground water is passed and throttled to 5.4
kg/cm2 in a steam separator. The dry steam produced in the separator is fed to double flow impulse
and reaction turbine with guaranteed engine efficiency of 85%. The turbine is directly coupledto a
3 phase, 60 Hz, 80% power factor, 13800-V air cooled generator. Exhaust is to be direct-contact
spray type main condenser designed to operate a vacuum of 647.5 mm Hg. Generator efficiency is
95%, ground water is 459 kg/s. What is the enthalpy of steam-water mixture in the steam
separator? Answer: A
A. 1038.10 kJ/kg
B. 1245.78 kJ/kg
C. 1543.25 kJ/kg
D. 1052.35 kJ/kg

6. Based on the data in problem No. 5, what is the enthalpy of steam entering the turbine? Answer: C
A. 2876.68 kJ/kg
B. 1874.67 kJ/kg
C. 2751.3 kJ/kg
D. 3574.4 kJ/kg

7. Based on the data in problem No. 5, what is the enthalpy of the exhaust steam? Answer: D
A. 2876.68 kJ.kg
B. 1876.67 kJ/kg
C. 2751.38 kJ/kg
D. 2204.15 kJ/kg
8. Based on the data in problem No. 5, what is the mass flow rate of steam entering the turbine?
Answer: A
A. 84.88 kg/s
B. 74.88 kg/s
C. 94.88 kg/s
D. 104.88 kg/s

9. Based on the data in problem No. 5, what is the actual power generated by the plant in MW?
Answer: A
A. 37.50 MW
B. 27.50 MW
C. 35.40 MW
D. 25.40 MW

10. Based on the data in problem No. 5, what is the apparent power developed by the generator in
KVA? Answer: D
A. 43,576.687 kVA
B. 51,874.678 kVA
C. 45,275.375 kVA
D. 46,877.475 kVA

11. Based on the data in problem No. 5, how many m 3/hr of fresh water that must be supplied to the
main condenser if water from cooling tower enters the condenser at 35 ºC and leaves at 48 ºC?
Answer: A
A. 11,105.393 m3/hr
B. 12,874.677 m3/hr
C. 22,751.377 m3/hr
D. 21,204.158 m3/hr

12. Based on the data in problem No. 5, how many production wells are needed to supply the plant
with pressurized water if each well has a capacity of 127,200 kg/hr? Answer: A
A. 13 wells
B. 14 wells
C. 12 wells
D. 10 wells

13. Based on the data in problem No. 5, determine the thermal efficiency of the geothermal plant?
Answer: D
A. 9.88%
B. 9.57%
C. 8.69%
D. 7.86%

14. In a geothermal power plant, the mass flow rate of ground water is 400 kg/s and the quality after
throttling is 20%. If the turbine power is 80 MW, what is the change in enthalpy of steam at the
inlet and outlet of the turbine? Answer: A
A. 1000 kJ/kg
B. 1500 kJ/kg
C. 2000 kJ/kg
D. 2500 kJ/kg
15. The mass flow rate of ground water in a 20 MW geothermal power plant is 150 kg/s and its
enthalpy is 1000 kJ/kg. if the quality after throttling is 28% and the over-all plant efficiency is
20%, what is the mass flow rate of steam entering the turbine? Answer: A
A. 28 kg/s
B. 76 kg/s
C. 18 kg/s
D. 24 kg/s

16. The turbine power of a geothermal plant is 2 MW and the enthalpy at the inlet of the turbine is
2500 kJ/kg. steam flows at the rate of 2.5 kg/s. if the enthalpy at the outlet of the condenser is 300
kJ/kg and the temperature rise of the cooling water is 10 ºC, what is the volume flow rate of the
cooling water? Answer: C
A. 109.37 m3/hr
B. 237.68 m3/hr
C. 120.37 m3/hr
D. 320.47 m3/hr

HYDROELECTRIC POWER PLANT


1. A hydraulic turbine receives water from a reservoir at an elevation of 100 m above it. What is the
minimum water flow in kg/s to produce a steady turbine output of 50 MW. Answer: B
A. 50, 247
B. 50, 968
C. 50, 672
D. 59, 465

2. A reaction turbine develops 500 BHP. Flow through the turbine is 50 cfs. Water enters at 20 fps
with a 100 ft pressure head. The elevation of the turbine above the tailwater level is 10 ft. find the
effective head. Answer: D
A. 130. 2 ft
B. 120. 2 ft
C. 110. 2 ft
D. 116. 2 ft

3. A hydro-electric plant discharges water at the rate of 0.75 m 3/s and enters the turbine at 0.35 mps
with a pressure of 275 kPa. Runner inside diameter is 550 mm, speed is 520 rpm and the turbine
efficiency is 88%. Find the turbine speed factor. Answer: A
A. 0. 638
B. 0. 386
C. 0. 836
D. 0. 386

4. A hydro-electric plant having 50 sq km reservoir area and 100 m dead head is used to generate
power. The energy utilized by the consumers whose load is connected to the power plant during a
five hour period is 13.5 x 10 6 kW-hr. The over-all generation efficiency is 75%. Find the fall in
the height of water in the reservoir after 5 hr period. Answer: B
A. 2. 13 m
B. 1. 32 m
C. 3. 21 m
D. 0. 53 m
5. A Pelton wheel is to be designed to run at 300 rpm under a effective head of 150 m. The ratio of
the nozzle diameter of the pitch circle is 1/12. Assuming efficiency of 84%, what is the size in m.
Assume a speed ratio of 0.45. Answer: C
A. 1.05
B. 2.00
C. 1.55
D. 2.86
6. 45 KW of the shaft power is developed by a turbine working under available head of 40 m. The
energy transferred from the water to the runner is 350 J. Assuming a mechanical efficiency of
95%, what is the discharge to the turbine in m3/s? Answer: B
A. 0.0345
B. 0.135
C. 1.511
D. 1.234

7. A vertical draft tube is installed on a Francis turbine and the total head to the center of the spiral
casing at the inlet is 38 m and velocity of water at the inlet is 5 m/s. The discharge is 2.1 m 3/s.
The hydraulic efficiency is 0.87 and over-all efficiency is 0.84. The velocities at the inlet and exit
of the draft tube are 5 m/s and 1.5 m/s respectively. The top of the draft tube is 1 m below the
center line of the spiral casing while the tailrace (water) level is 3 m from the top of the draft
tube. There is no velocity of whirl at either top or bottom of draft tube and leakage losses are
negligible. What is the power output of the turbine in KW? Answer: A
A. 748.8 kW
B. 632.9 kW
C. 901.3 kW
D. 832.6 Kw

8. A hydro-electric generating station is supplied from a reservoir of capacity 6,000,000 m 3 at a head


of 170 m. Assume hydraulic efficiency of 80% and electrical efficiency of 90%. The fall in the
reservoir level after a load of 15 MW has been supplied for 3 hrs, if the area of the reservoir is 2.5
km2 is closest to: Answer: A
A. 5.39 cm
B. 4.32 cm
C. 5.98 cm
D. 4.83 cm

9. An impulse turbine which has a diameter, D = 60 in. Speed N = 350 rpm, bucket angle B = 160
degrees, coefficient of velocity Cv = 0.98, relative speed φ = 45, a generator e m = 0.90, k = 0.90,
and a jet diameter from nozzle of 6 inches. Compute the power input in Hp. Answer: A
A. 2,862 HP
B. 2,512 HP
C. 2,933 HP
D. 2,366 HP

10. In Maria Christina Hydroelectric Project, the available head is 140 m, the water flow is one m 3
per second, efficiency of the turbine is 95%, efficiency of the generator is 95%, 3-phase, 60
cycles, voltage delivered is 4160 volts. Determine the estimated yearly income of the plant if cost
of the generated electric energy is P 0.60 per kW-hr. Answer: A
A. P 6,514,777.80
B. P 7,514,777.80
C. P 8,514,777.80
D. P 9,514,777.80

11. Water flows steadily with a velocity of 3.05 m/s in a horizontal pipe having a diameter of 15.24
cm. At one section of the pipe, the temperature and pressure of the water are 21 oC and 689.3 kPa
respectively. What is the friction factor? Answer: C
A. 0.156
B. 0.0091
C. 0.0185
D. 0.781

12. A hydroelectric plant has a 20 MW generator with the efficiency of 96%. The generator is directly
coupled to a vertical Francis type hydraulic turbine having an efficiency of 80%. The total gross
head on the turbine is 150 m while the loss of head due to friction is 4% of the gross head. The
runaway speed is not to exceed 750 rpm, determine the flow of water through the turbine in cfs.
Answer: C
A. 406.59
B. 5056.49
C. 650.94
D. 945.60

13. From the height of 65 m, water flows at the rate of 0.85 m 3/s and is driving a water turbine
connected to an electric generator revolving at 160 rpm. Calculate the power developed by the
turbine by KW if the total resisting torque due to friction is 540 N-m and the velocity of the water
leaving the turbine blades 4.75 m/s. Answer: A
A. 523.42
B. 543.21
C. 453.12
D. 435.21

14. A Pelton type turbine was installed 30 m below the head gate of the penstock. The head loss due
to friction is 15% of the given elevation. The length of the penstock is 80 m and the coefficient of
friction 0.00093. Determine the power output in KW. Answer: B
A. 14,292.74
B. 12,292.74
C. 19,242.74
D. 17,442.92

15. At a proposed hydro-electric plant site, the average elevation of the head of water is 600 m; the
tailwater elevation is 480 m. The average annual water flow is determined to be equal to that
volume flowing through a rectangular channel 4 m wide and 0.50 m deep and average velocity of
5.5 m/s. Assuming that the plant will operate 350 days per year, find the annual energy in kW-hr
that the plant site can develop if the hydraulic turbine that will be used has a efficiency of 80%
and generator efficiency of 92%. Consider a headwork loss of 4% of the available head. Answer:
B
A. 67,458,581
B. 76,854,851
C. 54,786,518
D. 48,576,185

16. A remote community in Mountain Province plans to put a small hydroelectric plant to service six
closely-located barangays estimated to consume 52,650,000 kW-hrs per annum. Expected flow of
water is 1665 m3/min. The most favourable location for the plant fixes the tailwater level at 480
m. The manufacturer of the turbine generator set have indicated the following performance data:
turbine efficiency 87%, generator efficiency 92%, loss in head work not exceed 3.8% of available
head. On order to pinpoint the most suitable area for the dam, determine the headwater elevation.
Answer: A
A. 508.76 m
B. 605.87 m
C. 750.08 m
D. 875.06 m

17. The difference in elevation between the surface of the water in a storage reservoir and the intake
to a turbine was 104 ft. during a test the pressure head at the latter point was 130 ft and the
discharge was 150 cfs. The inside diameter of the penstock was 35 inches. What was the
efficiency of the pipeline? Answer: B
A. 90.84%
B. 93.48%
C. 91.48%
D. 92.48%

18. In a hydraulic plant the difference in elevation between the surface of the water at intake and the
tailrace is 650 ft when the flow is 90 cfs, the friction loss in the penstock is 65 ft and the head
utilized by the turbine is 500 ft. The mechanical friction in the turbine is 110 Hp, and the leakage
loss is 4 cfs. Find the hydraulic efficiency. Answer: C
A. 87.45%
B. 84.57%
C. 85.47%
D. 78.54%

19. The difference in elevation between the source of the water supply and the centreline of the base
of the nozzle of a pelton wheel is 1300 ft. During the test the pressure at the end of the nozzle was
500 psig when the flow is 45 cfs. Inside diameter of penstock is 25 in. Compute the water
horsepower at the base of the nozzle. Answer: A
A. 5904.76 Hp
B. 4509.67 Hp
C. 7459.60 Hp
D. 6945.70 Hp

20. A Pelton wheel runs a constant speed under a head of 650 ft. The cross-sectional area of the jet is
0.50 ft2 and the nozzle friction loss is to be neglected. Suppose that the needle of the nozzle is to
be adjusted as to reduce the area of the jet from 0.50 ft 2 to 0.20 ft2. Under these conditions the
efficiency of the wheel is known to be 70%. Find the power output of the wheel. Answer: A
A. 2112.34 Hp
B. 2506.34 Hp
C. 3017.62 Hp
D. 3462.74 Hp

21. Water flows at the rate of 1.5 m 3/s from a height of 100 m which drive turbine connected to a 150
rpm generator. Determine the turbine brake power if the frictional torque is 600 N-m. Answer: B
A. 1862.02 kW
B. 1462.08 kW
C. 1642.08 kW
D. 1264.08 kW

22. The mechanical volumetric and total efficiencies of a turbine are 94%, 96% and 82%
respectively. Calculate the total head if the effective head is 50 m. Answer: A
A. 45.43 m
B. 44.35 m
C. 35.44 m
D. 53.44 m

23. A proposed hydro-electric power plant has the following data:


Elevation of normal headwater surface 194 m
Elevation of normal tailwater surface 60 m
Loss of head due to friction 6.5 m
Turbine discharge at full gate opening 5 m3/s
Turbine efficiency at rated capacity 90%
The turbine is to be connected to a 60 cycle AC generator. Calculate the turbine brakepower output.
Answer: D
A. 6852.5 kW
B. 6582.5 kW
C. 5268.5 kW
D. 5628.5 kW

24. A hydro-electric power plant consumes 60,000,000 KW-hr per year. What is the net head if the
expected flow is 1500 m3/min and over-all efficiency is 63%. Answer: C
A. 34.34 m
B. 43.43 m
C. 44.33 m
D. 33.44 m

25. A double overhung impulse turbine of 85% efficiency is direct connected to a 30,000 kW
generator of 93% efficiency. Elevation of the headwater level is 2730 ft above the sea level,
elevation of tailwater is 1260 ft above sea level, turbine setting is 12 ft, velocity of flow in
penstock is equal to 0.025√2gh, loss of head due to friction is 2% of gross head. Compute the
flow through the turbine in cfs. Answer: A
A. 157 ft3/s
B. 314 ft3/s
C. 413 ft3/s
D. 206.5 ft3/s
26. In the test of a certain turbine the pressure at the flange at the entrance to the spiral-turbine case
where the diameter is 30 in. was read by a mercury manometer. At a flow of 44 cfs the
manometer differential reading the U-tube was 9.541 ft Hg, the top of the lower mercury column
being 9.73 ft above the surface of the water in the tailrace, find the net head and the horsepower
developed by the turbine. Use turbine efficiency of 92%. Answer: A
A. 140.73 ft, 650 Hp
B. 170.43 ft, 650 Hp
C. 140.73 ft, 550 Hp
D. 170.43 ft, 550 Hp

27. River flows at 30 m 3/s and develops a total brake power of 5 MW. What is the available head if it
is proposed to install two turbines with 85% efficiency each? Answer: A
A. 19.99 m
B. 25.26 m
C. 16.75 m
D. 23.18 m

28. The total brake power developed by the two turbines is 6 MW. What is the capacity of the smaller
unit if one unit is thrice the capacity of the other? Answer: A
A. 1.5 MW
B. 2.0 MW
C. 2.5 MW
D. 3.0 MW

29. The over-all efficiency of a 10 MW hydro-electric plant is 85%. What is the secondary power
could this plant deliver if the actual power received by the costumer for that day is 100,000 kw-
hr? Answer: B
A. 110,000 kW-hrs
B. 104,000 kW-hrs
C. 124,000 kW-hrs
D. 130,000 kW-hrs

30. At a potential hydraulic-plant site the average elevation of the headwater is estimated at 1812 ft,
the tailwater elevation at 1575 ft. the average annual water flow was determined to be equal to
that volume flowing through a rectangular channel 30 ft wide with a 4 ft and an average velocity
of 15 ft/s. Find the annual electrical energy this site can produce water with a water turbine
efficiency of 85%, a generator efficiency of 95%, and the loss in the headworks equal to 3% of
the available head. Answer: B
A. 160,468,000 kW-hrs
B. 247,864,000 kW-hrs
C. 260,468,000 kW-hrs
D. 147,864,000 kW-hrs

31. A Pelton type turbine has a gross head of 40 m and a friction head loss of 6 m. What is the
penstock diameter if the length is 90 m and the coefficient of friction head loss is 0.001 (Morse)
Answer: A
A. 2040 mm
B. 3120 mm
C. 2440 mm
D. 2320 mm
32. The water velocity of a 5m x 1m channel is 6 m/s. What is the annual energy produced if the net
head is 120 m and the over-all efficiency is 80%. Answer: C
A. 494,247,258 kw-hrs
B. 247.497,582 kw-hrs
C. 247,494,528 kw-hrs
D. 472,497,582 kw-hrs

33. A 10,000 kW run-of-river hydro-electric plant has an available head of 200 ft and has an over-all
efficiency of 90% that remains constant for all loads. For a given day during the low water-steam
season the costumer receiving primary power have the following loads:
8000 kW for 2 hrs 4000 kW for 10 hrsfor 2 hr
2000 kW for 10 hrs
For a river flow remaining constant at 525 ft 3/s, how much secondary power could this plant
provide during the entire day? Answer: A

A. 104,220 kW-hrs
B. 220,104 kW-hrs
C. 204,220 kW-hrs
D. 120,104 kW-hrs

34. Find the number of generator poles for a generator running at 600 rpm and 60 Hz. Answer: A
A. 12
B. 24
C. 6
D. 48

35. The specific speed of the turbine is 85 rpm and running at 500 rpm. What is the maximum power
delivered by the generator if the head is 25 m and generator efficiency is 90%. Answer: B
A. 1,140.21 kW
B. 1,182.35 kW
C. 2,181.35 kW
D. 2,140.27 kW

36. Calculate the power that can be developed from a hydro-electric power plant of station having the
following data:
Catchment Area 90 km2
Average Annual Rainfall 120 cm
Run-off 85%
Available Head 350 m
Over-all Station 75%
Answer: D

A. 8746.06 kW
B. 6794.08 kW
C. 9746.80 kW
D. 7496.08 kW
37. In a hydroelectric power plant, the water surface on the crest of the dam is at elevation 75.3 m
while the water surface just at the outlet of the headgate is elevation 74.4 m. The headgate has 5
gates of 3ft x 3 ft leading to the penstock and are fully opened. Assume 61% as coefficient of
discharge, determine the quantity of water that enters the hydraulic turbine per second. Answer:
A
A. 464.5 ft3/s
B. 564.4 ft3/s
C. 644.5 ft3/s
D. 654.4 ft3/s

38. A hydro-electric impulse turbine is directly coupled to a 24 pole, 60 Hz alternator. It has a


specific speed of 60 rpm and develops 3000 Hp. What is the required diameter assuming a
peripheral speed ratio of 0.45 Answer: A
A. 0.661 m
B. 0.552 m
C. 0.443 m
D. 0.775 m

39. In a hydroelectric power plant the tailwater elevation is at 500 m. What is the head water
elevation if the net head is 30 m and the head loss is 5% of the gross head? Answer: C
A. 785.25 m
B. 582.57 m
C. 528.57 m
D. 758.25 m

40. A supply of water is available at a head of 60 m. it is proposed to build a turbine to operate at 125
rpm and this turbine is expected to develop 50 MW. What is the critical speed Answer: A
A. 193.79 rpm
B. 139.79 rpm
C. 391.97 rpm
D. 319.97 rpm

41. The elevation of the headwater is 3250 ft and that of the tailwater is 600 ft. penstock diameter is
30 in. water flowing 40 ft3/s, turbine efficiency 85%, friction factor taken as 0.035, turbine setting
is 8 ft above tailwater level. Penstock length 4000 ft. generator direct-connected to turbine has an
efficiency of 92%. Nozzle efficiency 96%. Compute the brake horsepower of the turbine Answer:
A
A. 9815 Hp
B. 8915 Hp
C. 5189 Hp
D. 9581 Hp
42. The flow of water at 50 m elevation is 45 m 3/s. if a hyro-electric plant is to be installed with
turbine efficiency of 80% and generator efficiency of 92%, what is the maximum power that the
plant could generate. Answer: B
A. 14.625 MW
B. 16.245 MW
C. 15.146 MW
D. 12.465 MW

43. Which of the following choices is a suitable generator speed for this type of runner with the
following data:
Horsepower 111,300
Net head 80 ft
Electrical efficiency 50 cycles/s. Answer: A
A. 53.57 rpm
B. 55.37 rpm
C. 57.53 rpm
D. 75.35 rpm

44. The difference in elevation between the source of the water supply and the centerline of the base
of the nozzle of a Pelton wheel is 500 m. during a test run, the pressure at the base of the nozzle
was 3 MPa when the flow was 2.0 m 3/s. Penstock inside diameter is 80 cm. determine the brake
power of the Pelton wheel if its efficiency is 85%. Answer: B
A. 311.46 kW
B. 5,113.46 kW
C. 4,115.34 kW
D. 4,511.43 kW

45. The pressure gage leading to the turbine casing of a Francis turbine reads 0.5 MPa and the center
of the spiral casing is 5 m above the tail race. What is the net head velocity of water entering the
turbine is am m/s? Answer: D
A. 67.01 m
B. 76.01 m
C. 70.61 m
D. 61.07 m

46. The tailwater and the headwater of a hydro-electric plant are 150 m and 200 m respectively. What
is the power if the flow is 15 m3/s and a head loss of 10% of the gross head? Answer: A
A. 6,621.75 kW
B. 7,621.65 kW
C. 5,621.76 kW
D. 4,621.56 kW

47. What is the specific speed of a turbine in a hyro-electric plant if the net head is 40 ft, brake power
is 3 MW and running at 500 rpm. Answer: A
A. 315.20 rpm
B. 513.02 rpm
C. 135.20 rpm
D. 351.20 rpm
48. A pelton wheel in a hydro-electric plant has an output of 17600 BHP at 300 rpm under a head of
510 m with 29 m3/s flow. Calculate its critical speed. Answer: B
A. 6.24 rpm
B. 3.72 rpm
C. 7.23 rpm
D. 2.73 rpm

49. In a hydro-electric plant, water flows at 10 m/s in a penstock of 1 m 2 cross- sectional area. If the
net head of the plant is 30 m and the turbine efficiency is 85%, what is the turbine output?
Answer: A
A. 2,501.55 kW
B. 2,100.21 kW
C. 3,626.34 kW
D. 3,124.65 kW

50. In a test of a reaction turbine the water flowing over the weir in the tailrace was found to be 38.8
ft3/s. the leakage into the tailrace was found to be 1 ft 3/s. the elevation of the centerline of the
shaft above the surface of the tailwater was 12.67 ft. the diameter of the turbine intake was 30 in.
and the pressure at this section was measured by a mercury U-tube. The readings in the two sides
of the mercury U-tube were 10.556 ft and 0.900 ft, the zero of the scale being at a level 3.82 ft
below that of the centerline of the turbine shaft. The generator output was 391.8 kW, friction and
windage loss 13.8kW,iron loss 2.0 kW and a mature loss of 4.4 kW. The specific gravity of the
mercury used was 13.57. calculate the efficiency of the turbine. Answer: A
A. 64.67%
B. 54.67%
C. 75.57%
D. 67.64%

51. A tank contains water. What is the intensity of pressure at a depth of 6 m? Answer: B
A. 68.0 kPA
B. 58.8 kPa
C. 78.0 kPa
D. 48.7 kPa

52. Convert water pressure of 50 kPAa in equivalent meter head of water. Answer: A
A. 5.1
B. 6.1
C. 7.1
D. 4.1

53. What is the over-all efficiency of the turbine if the power developed is 15,000 kW at a head of
170 meters and the nozzle discharge is 10 m3 a second? Answer: B
A. 70.093%
B. 89.980%
C. 79.989%
D. 90.096%
NON-CONVENTIONAL POWER PLANT

1. A solar collecting panel 20m 2 in area receives solar energy at the rate of 750 W/m 2. It is estimated
that 35% of the incident energy is lost to surrounding: water enters the panel at steady flow rate of
0.05kg/s at 15°C. Calculate the temperature of water leaving the collecting panel. Change in
elevation of water is 2m. Answer: A
A. 61.56°C
B. 45.76°C
C. 54.45°C
D. 71.45°C

2. An American type wind mill with an aerodynamic efficiency of 10% is wanted to pump
38liters/min from well to a tank located at 18.3m above the water surface of the well when the
wind has a velocity of 30kph. Consider that frictional losses of the system divert 30% of the
energy received by the wheel. What well diameter is needed? Answer: A
A. 2.5 m
B. 3.4 m
C. 1.5 m
D. 1.7 m

3. The sun generates an average of 2kW/m 2 when tapped as source of energy where in a flat plate
collector with an area of 1.5m 2 is used. Flow rate of water inside the collector is 40liter/hr.
calculate for the temperature rise of the working fluid in °C. Answer: A
A. 65.14°C
B. 60.15°C
C. 55.14°C
D. 50.15°C

4. A windmill with a 6m radius rotor is to be installed where atmospheric pressure conditions


prevail at 101.325kPa and 21°C. it was observed that wind is blowing steadily at an average
velocity of 20m/s. Assuming that the conversion efficiency is somewhere between 20-25%,
determine the maximum available power from the windmill. Answer: B
A. 124.78kW
B. 135.81kW
C. 167.25kW
D. 225.78kW

NUCLEAR POWER PLANT

1. A nuclear power plant is to have a capacity of 500MW-electrical. How many lbs of U 235 are
needed to operate the plant continuously for 6yrs if the plant capacity factor is 75% and thermal
efficiency 35%. Answer A
A. 5179.05
B. 5375.01
C. 6426.25
D. 6778.64
2. When a U235 nucleus fissions, about 250mev of energy are release. What fraction of the weight of
the nucleus is thereby converted into energy? Answer: B
A. 2.26 x 10-3
B. 1.13 x 10-3
C. 1.51 x 10-3
D. 2.52 x 10-3

3. A 600 MW PWR nuclear plant uses 4% enriched uranium fuel is a thermal efficiency of 36%.
The burn up allowed of the fissionable portion of the fuel is 20%. Calculate how many metric
tons of natural uranium is placed in the reactor vessel for 1yr continuous operation? Answer: B
A. 69.34
B. 79.35
C. 88.56
D. 56.77

4. Determine the wavelength of gamma photon having an energy of 1MEV. Answer: C


A. 1.53 x 10-10cm
B. 2.28 x 10-10cm
C. 1.24 x 10-10cm
D. 3.06 x 10-10cm

5. A nuclear power plant has a rated capacity of 200MW electrical power. The core contains 50tons
of uranium enriched to 2% U 235. Determine the average thermal neutron flux in the reactor
required at rated plant capacity. Answer: D
A. 1.18 x 1013 neutrons/cm2-s
B. 1.48 x 1013 neutrons/cm2-s
C. 1.28 x 1013 neutrons/cm2-s
D. 1.38 x 1013 neutrons/cm2-s

6. A proton has a kinetic energy of 10 Mev. What is its velocity? Answer: C


A. 4.34 x 1010 m/s
B. 3.34 x 1010m/s
C. 4.34 x 1010cm/s
D. 3.34 x 10 10cm/s

7. What is the wavelength of the most energetic x-rays from a 150,000 volt tube? Answer: A
A. 8.275 x 10-10cm
B. 8.275 x 10-10cm
C. 7.825 x 10-10cm
D. 7.852 x 10-10cm

8. If all the exact isotopic mass of He4 is 4.00187 amu, calculate the total binding energy and the
binding energy per nucleon. Answer: B
A. 30.14 Mev, 8.53Mev/nucleon
B. 30.14 Mev, 7.53Mev/nucleon
C. 35.41 Mev, 7.53Mev/nucleon
D. 35.41 Mev, 8.53Mev/nucleon
MACHINE FOUNDATION

1. What is the required base area of the foundation to support an engine with specified speed of
1200 rpm and weight of 9000kg. assume a safe bearing capacity of soil as 47.867kPa. use c=0.11.
Answer: B
A. 5.57m2
B. 8.87m2
C. 7.75m2
D. 10.5m2

2. A 1:2:3 mix of concrete with 6 gallons of water per bag cement is being prepared. Determine the
weight of 1 ft3 of finished concrete of this mixture. The materials to used have the following
characteristics:
1 bag cement (1ft3) weighs 94 lbs
1 ft3 sand weighs 105 lbs
1 ft3 of stone weighs 105 lbs
Specific gravity of cement 3.1
Specific gravity of sand 2.7
Specific gravity of broken stone 2.7
Answer: D
A. 164 lbs/ft3
B. 172 lbs/ft3
C. 162 lbs/ft3
D. 152 lbs/ft3

3. What is the length of the machine foundation if the length of the bedplate is 4m? Answer: A
A. 4.8m
B. 5.5m
C. 3.3m
D. 6.6m

4. If the width of the bedplate is 3m and the clearance is 0.3 what is the upper width of the machine
foundation? Answer: C
A. 5.5m
B. 4.3m
C. 3.3m
D. 5.5m

5. What is the approximate weight of the engine if the weight of the machine foundation is 35000
kg? Answer: C
A. 5870 kg
B. 5750 kg
C. 8570 kg
D. 7850 kg

6. Calculate the cu. yard of stone required for a concrete mixture of 1:2:4 of a foundation with
dimension 4m x 5m x 6m? Answer: B
A. 129.61 yard3
B. 138.14 yard3
C. 156.82 yard3
D. 167.99 yard3
7. If the volume of the foundation is 5cu. ft and the density of concrete is 160 lbs/cu.ft what is the
total weight? Answer: C
A. 600lbs
B. 700lbs
C. 800lbs
D. 900lbs

8. The bedplate dimension of a rectangular foundation cross-section is 2m x 12m. the height of


foundation and the uniform clearance of each are one meter and 0.50m respectively. Find the
weight of steel bars if the weight of the steel bar reinforcements needed is ¾% of the weight of
the foundation. Note: use density of concrete equals 2406kg/m 3. Answer: A
A. 703.75kg
B. 806.10kg
C. 644.54kg
D. 509.92kg

9. Piles are driven in a quicksand (Bearing = 5tons/m 2) on 900mm centers. They are driven until the
penetration under the last hammer blow is 25mm. the concrete slab 900mm thick. What is the
average bearing power in tons per m2 does this foundation provide? Answer: B
A. 12.81
B. 10.86
C. 11.41
D. 13.77

CHIMNEY

1. A steam boiler plant consumes 9000 kg of coal per hr and produces 20 kg of dry flue gasses per
kg of coal fired. Outside air temperature 32°C, average temperature of the flue gas entering the
chimney is 343°C and the average temperature of dry flue gas of the chimney is 260°C. The gage
fluid density is 994.78 kg/m2 and the theoretical draft of 2.286cm of water at the chimney base is
needed when the barometric pressure is 760mm Hg. Determine the height of the chimney?
Answer: C
A. 30
B. 40
C. 50
D. 60

2. If the actual draft for a furnace is 6.239 cm of water the frictional losses of the stack are 15% of
the theoretical draft, calculate the required stack height in meters. Assume that the flue gas have
an average temperature of 149°C and molecular weight of 30. Assume air temperature of 21°C.
Answer: C
A. 215
B. 230
C. 220
D. 210
3. A steam generator with economizer and air heater has an overall draft loss of 21.78cm of water. If
the stack gasses are at 177°C and if the atmosphere is at 101.3kPa and 26°C. what is the
theoretical height of the stack in meter is needed when no draft fans are used? Assume that the
gas constant for the flue gasses is the same as that for air. Answer: D
A. 565
B. 535
C. 545
D. 550

4. A coal fired steam boiler uses 3000 kg of coal per hour. Air required for combustion is 15.5 kg
per kg coal at barometric pressure of 98.2 kPa. The flue gas has a temperature of 285°C and an
average molecular weight of 30. Assuming an ash loss of 11% and allowable gas velocity of
7.5m/s, find the diameter of the chimney. Answer: A
A. 1.91 m
B. 1.82 m
C. 2.62 m
D. 2.93 m

5. The over-all draft loss of steam generating unit is 400 mm water. Air enters at 101.325kPa, 26°C
and the average flue gas temperature is found to be 250°C. if no draft fans are to be installed,
what is the height of the chimney? Assume Rg=0.227kJ/kg-K. Answer: A
A. 831.60 m
B. 722.58 m
C. 926.65 m
D. 599.11 m

6. What is the height of the chimney if the driving pressure is 30 Pa and the gas and air densities are
1kg/m3 and 1.5 kg/m3 respectively? Answer: C
A. 50.82 m
B. 71.16 m
C. 61.16 m
D. 80.22 m

7. If the air required for combustion is 20 kg per kg of coal and the boiler uses 3000 kg of coal per
hr, determine the mass of gas entering the chimney. Assume an ash loss of 15%. Answer: C
A. 40644kg/hr
B. 70200kg/hr
C. 62550kg/hr
D. 50500kg/hr

8. A 15 kg gas enters a chimney at 10m/s. if the temperature and pressure of a gas are 26°C and 100
kPa respectively, what is the diameter of the chimney. Use R=0.287kJ/kg-K. Answer: D
A. 1.57 m
B. 2.65 m
C. 2.22 m
D. 1.28 m
9. A 39.5 m high chimney of radial brick masonry is described by the following top and bottom
dimensions. D2= 1.9m, d2= 1.5m, D1= 3.2m, d1= 2.3m. Determine the moment due to wind
load. Answer: D
A. 172051 kg-m
B. 160338 kg-m
C. 150160 kg-m
D. 182030 kg-m

10. In a steam power plant, 5 kg of coal is consumed per second and it was late found that 25 kg of
dry flue gas is produced per kg of coal fired. Air enters at 25°C. The average temperature of the
flue gas entering the chimney is 250°C. A theoretical draft of 5 cm of water at the base of the
chimney is needed. The gage fluid specific volume is 0.0025 m 3/kg. If the molecular weight of
the gas is 20, calculate the height of chimney. Answer: A
A. 41.15 m
B. 36.27 m
C. 52.86 m
D. 55.42 m

11. A power plant is situated at an altitude having an ambient air at 96.53kPa and 23.88°C. Flue gases
at a rate of 5kg/s enters the stack at 200°C and leaves at 160°C. The flow gases gravimetric
analysis are 18% CO2, 7% O2, and 75% N2. Note: The actual velocity is 40% of the theoretical
velocity. Answer: C
A. 0.75 m
B. 0.85 m
C. 0.95 m
D. 1.15 m

VARIABLE LOAD PROBLEMS

1. A central station is supplying energy to a community through two sub-stations. One substation
feeds four distributing circuits; the other is six. The maximum daily recorded demands are:
POWER STATION 12000 kW
Substation A 6000 kW
Feeder 1 1700 kW
Feeder 2 1800 kW
Feeder 3 2800 kW
Feeder 4 600 kW
Substation B 9000 kW
Feeder 1 620 kW
Feeder 2 1500 kW
Feeder 3 1000 kW
Feeder 4 2900 kW
Feeder 5 2200 kW
Feeder 6 3300 kW
Calculate the diversity factor between substations, between feeders on Substation A and between
feeders on Substation B. Answer: A
A. 1.25, 1.15, 1.25
B. 1.36. 1.21, 1.36
C. 1.28, 1.19, 1.32
D. 1.32, 1.17. 1.35
2. A 75 MW power plant has an average load of 35000kW and a load factor of 65%. Find the
reserve over peak. Answer: A
A. 21.25 MW
B. 23.41 MW
C. 25.38 MW
D. 18.75 MW

3. The annual peak load on a 15000 kW power plant is 10500kW. Two substation are supplied by
this plant. Annual energy dispatched though substation A is 27,500,000 kW-hr with a peak load at
8900 kW, 16,500,000 are sent through substation B with a peak load at 6650kW. Neglect line
losses. Find the diversity factor between substation and capacity factor of the power plant.
Answer: B
A. 1.48, 0.446
B. 1.48, 0.335
C. 1.75, 0.335
D. 1.75, 0.446

4. What is the daily average load in a certain power plant if the daily energy produced is 500,000
kW-hr. Answer: D
A. 28.29 kW
B. 30.26 kW
C. 19.61 kW
D. 20.83 kW

5. A distributing transformers supplies a group of general power customers having a connected load
of 186kW. Demand factor and diversity factor are 0.75 and 1.5 respectively. If the load factor for
the group will average of 45% and the energy sells 3.5 cents per kW-hr, what will be the monthly
income (30-day) from energy delivered through this transformer? Assume energy motor
efficiency is 75%. Answer: A
A. P 1406.16
B. P 2812.32
C. P 1502.61
D. P 3003.22

6. What is the annual capacity factor of the plant if the annul energy produced in a 150MW power
plant is 500,000,000 kW-hrs? Answer: A
A. 38.05%
B. 44.04%
C. 56.785%
D. 34.44%

7. A power plant is said to have/had a use factor of 48.5% and a capacity factor of 42.4%. How any
hours did it operate during the year? Answer: B
A. 6600.32 hrs
B. 7658.23 hrs
C. 8600.32 hrs
D. 5658.23 hrs
8. A central station has an annual factors as follows: Load factor 58.5%, capacity factor 40.90%, use
factor 45.2%. The reversed carried over and above the peak load is 8900 kW. Find the no. of hrs
per year not in service. Answer: A
A. 833.37 hrs
B. 733.38 hrs
C. 783.33 hrs
D. 873.33 hrs

9. A 50,000 kW steam plant delivers an annual output of 238,000,000 kW-hr with a peak load of
42,860kW. What is the annual load factor and capacity factor? Answer: C
A. 0.634, 0.534
B. 0.643, 0.534
C. 0.634, 0.543
D. 0.643, 0.543

10. Calculate the use factor of a power plant is the capacity factor is 35% ad it operates 8000 hrs
during the year? Answer: A
A. 38.325%
B. 33.825%
C. 35.832%
D. 35.538%

11. Given a load factor, 0.48, installed capacity, 35000 kW, reverse over peak 3000 kW, hours out of
service per year 410. Find the capacity and the use factor. Answer: A
A. 48.39%, 46.04%
B. 43.89%, 44.06%
C. 43.89%, 46.04%
D. 48.39%, 44.06%
12. The system shown in the figure consists in part of transformer serving costumers X, Y, Z.
Estimate the peak load on the transformer.
X – store building with 5kW lightning, 25kW small motor power
Y – store building with 18kW lightning, 35kW small motor power
Z – store building with 55kW lightning, 80kW large motor power

Distribution
Transformer

X Y Z

Lighting = 5 kW Lighting = 18 kW Lighting = 55kW


Motor = 25 kW Motor = 35 kW Motor = 80 kW

Answer: B
A. 75.08kW
B. 85.07kW
C. 95.84kW
D. 105.76kW

13. A daily load curve which exhibited a 15 minute peak of 150,000kW is drawn to scales of 1 cm =
3hrs and 1 cm = 10000kW. The area under the curve is measured by a planimeter and fund to be
60 cm2. What is the load factor based on the 15 minute peak? Answer: A
A. 0.50
B. 0.75
C. 0.55
D. 0.65

14. A distributing transformer supplies a group of general power costumers having a connected load
of 200kW. If the demand and diversity factor are 0.75 and 1.5 respectively. Calculate the
maximum simultaneous demands. Answer: C
A. 150 kW
B. 240 kW
C. 100 kW
D. 200 kW

15. If the rated capacity of the system is 50,000 kW and the actual maximum demand is 35,000 kW,
what is the utilization factor? Answer: B
A. 0.60
B. 0.70
C. 0.50
D. 0.80
PIPING SYSTEM

1. Water flows steadily with a velocity of 3.05m/s in a horizontal pipe having a diameter of 15.24
cm. At one section of the pipe, the temperature and pressure of the water are 21°C and 689.13kPa,
respectively. At a distance of 354.8m downstream, the pressure is 516.9kPa. What is the friction
factor? Answer: C
A. 0.134
B. 0.005
C. 0.0189
D. 0.641

2. Find the kg/m2 fiction loss in a 30.5 m copper pipe, 19.1 mm I.D. in which there is a flow of 544
kg/hr of 24°Be fuel oil at 21.1°C. Answer: A
A. 0.635 kg/cm2
B. 0.365 kg/cm2
C. 0.563 kg/cm2
D. 0.536 kg/cm2

3. A certain 18.30 m section of insulated 234 mm schedule 80 steel pipe when warmed to operating
temperature from a cold state of 4.4°C produces initial condensation which a trap should drain
out as the line heats up. Assume this takes 5minutes. Steam saturated at 28.1kg/cm 2 gage.
Estimate the discharge capacity of the trap in kg/hr. The pipe weighs 96.04kg/m. Answer: B
A. 2152.34 kg/hr
B. 1196.52 kg/hr
C. 2196.52 kg/hr
D. 1152.34 kg/hr

4. What is the working stress of a schedule 120 pipe with the following data:
Outside diameter 219.08mm
Wall thickness 18.24 mm
Working pressure 17.60kg/cm2 gage
Allowance corrosion 1.65
Answer: A
A. 109.17 kg/cm2
B. 190.17 kg/cm2
C. 160.39 kg/cm2
D. 140.39 kg/cm2

5. A 254mm schedule 80 A-53 steel pipe is clamped between two rigid supports 6.1 apart when its
temperature is 15.60°C. a liquid at 93.3°C is then pumped through the line. What is the area of
the material under stress? The outside diameter and the wall thickness of the pipe are 273.05mm
and 15.06 mm respectively. Answer: D
A. 0.1650 m2
B. 0.0165 m2
C. 0.1220 m2
D. 0.0122 m2
Miscellaneous ME Board Problems

6. The over-all efficiency of an electric system, coal pile to lights, is 11%. What is the fraction of a
kg of coal containing 30250 kJ/kg must be consumed to light a 100W lamp for 45 minutes?
Answer: C
A. 0.78 kg
B. 0.095 kg
C. 0.081 kg
D. 0.065 kg

7. A power plant serves a factory having two 22 kW motors and ten 3.7 kW motors. Assume the
efficiency of motors 80%, of transmission line 95%, of generator 92%. What should be the rated
capacity of the engine? Answer: A
A. 115.85 kW
B. 200.75 kW
C. 245.56 kW
D. 457.35 kW

8. An electric motor converted 700 W of electrical input into work at 58.186 kg-m/s. The speed was
1750rpm. Find the kg-m of driven torque. Answer: C
A. 0.183 kg-m
B. 0.382 kg-m
C. 0.318 kg-m
D. 0.138 kg-m

9. In a charge over from steam to electric heating, a unit which had been considering 4.5 kg dry and
saturated steam at 2.109 kg/cm 2 absolute each 15 min is to be electrically heated from 220V
circuit. How many 12 ohm resistor in parallel would be needed to supply the same heat? Latent
heat of vaporization at 2.109 kg/cm2 is 2198.535 kJ/kg. Answer: C
A. 3
B. 2
C. 4
D. 5

10. An electric heater is to heat 11 kg of oil per minute from 4.5°C to 65.5°C. Specific heat of the oil
is 2.1 kJ/kg°C. How many watts should this heater consume? Answer: B
A. 24.385 kW
B. 23.485 kW
C. 28.385 kW
D. 32.854 kW

11. The flow of a river is 4.25m3/s at a site where a 22.86m hydrostatic head can be created by the
erection of the dam. What is the potential capacity of the hydro-electric power plant if installed at
this site? Consider that the hydraulic efficiency of energy conversion can be 80%, and that the
electric efficiency can be 90%. Answer: B
A. 866.522 kW
B. 686.225 kW
C. 566.822 kW
D. 656.522 kW
12. A hoist is to raise 11147.727 N mine cage at the rate of 4.57 m/s. Mechanical efficiency of the
hoist 92%. What is the kW required to drive at this speed? Answer: C
A. 73.533 kW
B. 73.335 kW
C. 55.375 kW
D. 53.375 kW

13. A turbo generator rotating mass has a moment of inertia of 555 hyls/m 2. It is delivering 2500 kW
at 1800rpm. The load then suddenly increased to 2550 kW, the developed steam power remaining
unchanged. What is the resulting speed in rpm in 10 seconds? Answer: A
A. 177.82 rpm
B. 182.77 rpm
C. 128.77 rpm
D. 217.87 rpm

14. Forty percent of the electrical input to a motor driven pump is converted into a hydraulic jet
0.013m in diameter for the purpose of wasting down ashes. Find the het velocity inm/s. The
motor has a 3-phase,220 V, 7.5 amp rating. Power factor 85%. Answer: A
A. 52.38 m/s
B. 58.32 m/s
C. 38.52 m/s
D. 35.83 m/s

15. The difference in tension between the sides if a belt burning over a 0.762m diameter pulley is
22.727 kg. Pulley speed is 500 rpm. What is kW transmitted? Answer: A
A. 4.45 kW
B. 3.45 kW
C. 3.54 kW
D. 6.44 kW

16. What is the power of the steam jet 0.015 m in diameter moving at 761.962 m/s? Steam condition,
1.406 kg/cm2 dry and saturated (v1 = 0.19m3/kg). Answer: C
A. 220.27kW
B. 202.78kW
C. 208.72kW
D. 280.27kW

17. A pump is lifting water through 5.486 m to fill a 566.254 m3 tank. The over-all energy efficiency
is 80%. Calculate the length of time that 7.46 kW applied to the pump will require to complete
the job. Answer: B
A. 85.11 min
B. 81.51 min
C. 75.11 min
D. 71.51 min

Potrebbero piacerti anche